MH exam 2_ chs.6-9

Lakukan tugas rumah & ujian kamu dengan baik sekarang menggunakan Quizwiz!

Which statement made by the nurse concerning a client experiencing musculoskeletal pain demonstrates attention to the evaluation portion of the nursing process? "The client's daughter confirmed that he had knee replacement surgery 3 years ago." "The client's inability to ambulate affectively without assistance is a priority problem." "After 2 weeks of physical therapy, the client can safely walk the length of the hallway." "The client has expressed a strong fear of falling when asked to walk without assistance."

"After 2 weeks of physical therapy, the client can safely walk the length of the hallway." The nursing process includes assessment, nursing diagnosis, outcome identification, planning, implementation, and evaluation. Stating that the client is now able to safely ambulate the length of the hallway demonstrates an expected outcome. The nurse stating the client's daughter confirmed that the client had a knee replacement surgery 3 years ago is incorrect, because it demonstrates nursing assessment. The nurse stating that the client is unable to ambulate effectively is incorrect, because it demonstrates nursing diagnosis. The nurse stating that the client expressed a fear of falling when asked to walk without assistance is incorrect, because it demonstrates nursing diagnosis. p. 76

An elderly client recently admitted to the long term care facility for rehabilitation tells the nurse, "No one cares; instead of helping me, they sent me here to die." Which would be the most effective response by the nurse? "Do you feel that no one cares?" "You know that's not true, your family cares." "You will only be here a short time so please don't be this way." "Let's not worry about that right now, let's get you admitted and to therapy."

"Do you feel that no one cares?" Restating the main idea expressed allows the client to clarify if that was not truly the intent. Stating that the client knows this to be untrue minimizes the client's feelings and is not therapeutic. Telling the client that he or she will be in the facility only for a short term is false reassurance and is not therapeutic. Changing the subject by stating, "Let's not worry about that and get you to therapy" is a nontherapeutic response that may invalidate the client's feelings. p. 98

After work, a psychiatric nurse makes several comments to a family member. Which comment indicates that this nurse needs clinical supervision? "It seems like my work load is very heavy right now." "My supervisor announced plans for renovations to our offices." "I am taking care of a prominent political person from our community." "We have a new psychiatrist with expertise in working with troubled veterans."

"I am taking care of a prominent political person from our community." The correct response indicates that the nurse has attached an additional importance to care of one client (a politician). In some instances, this comment also could jeopardize the client's confidentiality. Supervision by a more experienced clinician or team member is essential to developing one's competence. Nurses should consistently practice self-care by sharing their own feelings with others; however, this sharing must respect professional boundaries. Indicating a work load is heavy, that a supervisor announced plans for renovation, or that there is a new expert for a given population are all acceptable comments to share with family members and do not warrant supervision. Test-Taking Tip: Look for answers that focus on the client or are directed toward feelings. p. 111

Which statement made by a mental health nurse demonstrates the need for further education regarding therapeutic communication techniques? "When I use therapeutic silence, I'm giving the client time to think and reflect." "Sharing perceptions doesn't mean I tell the client how my experiences are similar to his or hers." "I generally find it helpful to ask the client why he or she blames others for the mistakes he or she has made." "It's not therapeutic to give the client suggestions as to what he or she needs to do to fix his or her problems."

"I generally find it helpful to ask the client why he or she blames others for the mistakes he or she has made." Asking why the client is behaving in a particular manner often is viewed as judgmental by the client. Presenting such an attitude would be a barrier to communication and thus nontherapeutic. Stating, "When I use therapeutic silence, I'm giving the client time to think and reflect" describes an effective use of silence. Sharing perceptions is used to clarify an understanding of what the client is thinking or feeling. Suggestions are therapeutic only when given as possible alternatives for the client to consider, not when given as advice. Test-Taking Tip: Note the number of questions and the total time allotted for the test to calculate the times at which you should be halfway and three-quarters finished with the test. Look at the clock only every 10 minutes or so. p. 120

The home care nurse is making an initial visit to provide support and education to the family of a child with severe asthma. During the assessment, the mother steps out of the room and returns smelling of cigarette smoke. Which would be the most therapeutic way to address this? "You really need to stop smoking if you value the life of your child." "I noticed that you stepped out to smoke, do you see this as an issue with your child's asthma?" "Do you realize that your refusal to stop smoking may result in rehospitalization of your child?" "Why do you continue to smoke when your child has such severe asthma? I can smell it on your clothes."

"I noticed that you stepped out to smoke, do you see this as an issue with your child's asthma?" Stating that the nurse noticed that the individual stepped outside to smoke is making an observation which can call attention to a person's behavior and help them describe thoughts and feelings related to the behavior. Using fear or threats such as stating that "you really need to stop smoking if you value the child's life" or that the behavior will result in rehospitalization is not therapeutic and implies judgment. "Why" questions imply criticism and are not therapeutic. p. 99

In an inpatient psychiatric unit, an aggressive client frequently insults a timid client during group sessions. In the group session today, the client spoke out to the bully about the behavior. Which statement by the nurse is most therapeutic to show approval for the client's actions? "You really shouldn't have addressed the bully in the group session today." "I hope that you will be able to defend yourself again when you are bullied." "You did a great job of defending yourself today in group and standing up to the bully." "I noticed you addressed the insulting behavior today in group. How did It feel to be more assertive?"

"I noticed you addressed the insulting behavior today in group. How did It feel to be more assertive?" Asking the client how they felt about being more assertive is correct because it opens the way for finding out if the client was scared or comfortable, wants to work more on assertiveness, or has other issues to discuss. Telling the client they did a great job is incorrect because it is giving approval and stops further communication. Telling the client he or she should not have addressed the bully is incorrect, because it is disapproving. Telling the client he or she will be able to defend him- or herself again when being bullied is incorrect because it does not provide the opportunity for further discussion. p. 98

Which question asked by the nurse would be considered seeking clarification with a client who thinks that he or she are being watched by the government? "You feel that you are being watched?" "Can you describe who you think is watching you? "I am here to take care of you today, do you have any questions before we start? "I'm unclear, could you give me an example of when you thought you were being watched?"

"I'm unclear, could you give me an example of when you thought you were being watched?" When the nurse states he or she is unclear, the nurse is using the technique of seeking clarification from the client. Repeating the main idea being expressed such as being watched is use of the technique of restating. Asking the client to describe who is watching them would be the use of the technique exploring. Providing facts the person may need such as stating the role of the nurse is the technique of giving information. p. 97

Which question will the nurse ask the client during a psychosocial assessment to determine existing social patterns? "Please describe your typical day." "How do you spend your free time?" "Do you have a religious affiliation?" "With whom do you talk to when you are upset?"

"Please describe your typical day." To gather information about the client's social patterns, the nurse would ask him or her to describe a typical day. Asking the client how his or her free time is spent is incorrect, because it is associated with interests and abilities. Asking the client if he or she has a religious affiliation is incorrect, because it is associated with the spiritual aspect of the psychosocial assessment. Asking the client with whom he or she talks to when the client is upset is incorrect, because it is a part of the coping abilities portion of the psychosocial assessment. Test-Taking Tip: Look for answers that focus on the patient or are directed toward feelings. p. 81

A single young adult who has had a 10-week pregnancy recently confirmed asks the nurse what she should do because she has no money, no insurance, and the father is not interested in being involved. She tells the nurse that she truly does not want to raise a child. Which response by the nurse would be nontherapeutic? -"What are some ways that you think you could deal with this situation?" -"If you truly do not want the child, adoption or abortion may be the best alternatives." -"You think the father of the baby is not interested in being involved in the pregnancy?" -"What did it mean to you when the father of the baby told you that he did not want to be involved?"

"If you truly do not want the child, adoption or abortion may be the best alternatives." Offering advice such as telling the client her choices are abortion or adoption is not helpful and does not allow the client to make their own decision, and is thus nontherapeutic. Encouraging the client to think of other ways she could deal with the situation encourages her to identify alternative actions and is a therapeutic response. Asking if the client thinks the father of the baby is not interested in being involved in the pregnancy uses the technique of reflecting, which can be effective in directing questions back to the client. Asking the client what it meant to her when the father of the baby told them he did not want to be involved is the technique of evaluation, which encourages her to explore the situation from her own perspective. Test-Taking Tip: Multiple choice questions can be challenging, because students think they will recognize the right answer when they see it or that the right answer will somehow stand out from the other choices. This is a dangerous misconception. The more carefully the question is constructed, the more each of the choices will seem like the correct response. p. 99

The home care nurse is visiting a client who is managing congestive heart failure at home. The client has gained 3 pounds in the past few days, has 2+ edema of the lower extremities, and is having difficulty breathing. Which statement by the nurse would be considered nontherapeutic as the nurse attempts to determine the factors that have led to the current assessment findings? "I notice that you are having trouble breathing." "When did you first notice the trouble breathing and the weight gain?" "It seems you quit taking your medications, why did you stop taking them?" "Perhaps you and I can work together to determine what has led to these symptoms."

"It seems you quit taking your medications, why did you stop taking them?" The nurse's statement that the client quit taking the medications and asking "why" makes a judgment and may place the client on the defensive; thus this would be a nontherapeutic statement. Asking when the client first noticed the issues helps to place the events in a sequence and can be helpful to determine what may have created the current issues. Suggesting that the nurse and client can work together indicates a desire to collaborate, which is a therapeutic approach to the situation. p. 99

A client diagnosed with paranoid schizophrenia tells the nurse, "I'm here on a top secret mission for the President. Don't tell anyone I am here." Which response should the nurse provide to address the client's primary concern? "Let's talk about something other than your mission for the President." "Your admission papers do not list you as an employee of the President." "You have lost touch with reality, which is a symptom of your illness." "It sounds like you have some concerns about your privacy. You are safe here."

"It sounds like you have some concerns about your privacy. You are safe here." The correct response focuses on the client's perception and feelings that it is important no one knows where he or she is. Reassuring the client of his or her privacy also offers a safe environment. Changing the subject takes the focus of the interview off the client's feelings and minimizes his or her experiences. It is important not to challenge the client's beliefs by noting the admission papers or saying the client has lost touch with reality, even if the beliefs are unrealistic. Challenging undermines the client's trust in the nurse. The nurse should try to understand the underlying feelings or thoughts that the client's message conveys rather than focusing on the accuracy of the unrealistic statement itself. Test-Taking Tip: Avoid spending excessive time on any one question. Most questions can be answered in one to two minutes. p. 120

A client diagnosed with major depressive disorder says to the nurse, "You're assigned to take care of me, but you keep talking to other clients. You need to spend more time with me." Which is the nurse's most therapeutic response? "It sounds like you're saying you need more attention." "I have other clients whose needs are greater than yours." "My time is equally divided among all clients assigned to me." "Thank you, but I will decide how much time to spend with my clients."

"It sounds like you're saying you need more attention." Telling the client it sounds like he or she is expressing a need for more attention demonstrates the therapeutic technique of reflecting. This encourages the client to accept his or her own feelings. Responses that other clients have greater needs, that the nurse divides time equally, and that the nurse will decide how to spend his or her time might address the nurse's situation, but they are not therapeutic for the client, in that they do not address the client's viewpoint or feelings in a way that encourages growth and understanding. p. 97

A client is hospitalized after making a suicide attempt when his or her spouse asks for a divorce. Which comment by the nurse is therapeutic for this client? "Don't you think your life is more valuable than an unhappy marriage?" "You should forget about your marriage and move on with your life." "Let's consider ways other than suicide to cope with your feelings." "I understand your depression. When I got divorced, I was overwhelmed too."

"Let's consider ways other than suicide to cope with your feelings." Considering coping mechanisms other than suicide demonstrates one of the working phase tasks of promoting practice of alternative adaptive behaviors. It also keeps the focus on the client's perception and feelings. Questioning the client's values is nontherapeutic. Telling the client to forget the divorce is unrealistic and minimizes the client's feelings. Bringing up the nurse's own divorce takes the focus off the client. Test-Taking Tip: Come to your test prep with a positive attitude about yourself, your knowledge, and your test-taking abilities. A positive attitude is achieved through self-confidence gained by effective study. This means (a) answering questions (assessment), (b) organizing study time (planning), (c) reading and further study (implementation), and (d) answering questions (evaluation). p. 116

How should the nurse respond to a client who reports thinking of dropping out of college because it is too stressful? "Don't let them beat you! Fight back!" "School is stressful. What do you find most stressful?" "I know just what you are going through. The stress is terrible." "You have only two more semesters. You will be glad if you stick it out."

"School is stressful. What do you find most stressful?" This response acknowledges the client's perception of school as difficult and asks for further information, keeping the interview focused on the client and his or her feelings. This response suggests the nurse is listening actively and is concerned. The nurse should avoid attacking the target of the client's anxiety. Saying the nurse knows what the client is going through minimizes the client's experience. Saying "the stress is terrible" is an interpretation that should be avoided; this takes the focus off the client's perception. The nurse should also avoid giving advice or false reassurance. p. 120

The nurse is discussing the upcoming transfer of a client to a long term care facility with the client's spouse who is struggling with the decision for long-term care. The spouse asks the nurse if he or she has done the right thing. Which statement would be most therapeutic in this situation? "Yes, you have done all that you can, this nursing home will provide good care." "Tell me more about how you feel about your spouse going into long-term care." "Everything will be all right; your spouse may not need to be there long anyway." "I'm glad you have made this decision; even though it was a hard one, it is the right one."

"Tell me more about how you feel about your spouse going into long-term care." Asking the spouse to talk more about his or her feelings allows reflection and supports decision making. Stating that the spouse has done everything he or she can, made the right decision, or that everything will be all right can minimize the person's feelings and be judgmental on the part of the nurse. p. 99

Which statement best reflects the nurse's use of exploring in a conversation? "Tell me more about the accident." "Have you had an accident before?" "Why do you think this accident happened?" "How could you prevent a similar accident in the future?"

"Tell me more about the accident." Encouraging the client to tell the nurse more about the accident encourages exploration of the issue. Asking if the client has had an accident before encourages comparison. Asking how the client could prevent accidents in the future encourages formulation of a plan of action. "Why" questions can put clients on the defensive and are nontherapeutic. Test-Taking Tip: After you have eliminated one or more choices, you may discover that two of the options are very similar. This can be helpful, because it may mean that one of these look-alike answers is the best choice and the other is a very good distractor. Test both of these options against the stem. Ask yourself which one completes the incomplete statement grammatically and which one answers the question more fully and completely. The option that best completes or answers the stem is the one you should choose. Here, too, pause for a few seconds, give your brain time to reflect, and recall may occur. p. 97

A client diagnosed with paranoid delusions tells the nurse, "Do you see Jesus standing over there? He is right in front of the window." Which response by the nurse would be the most therapeutic? "I do see someone that looks a bit like Jesus by the window." "What makes you think Jesus is standing over by the window?" "That is one of the therapists standing in front of the window, not Jesus." "That is just another one of your delusions, let's find something to distract you."

"That is one of the therapists standing in front of the window, not Jesus." Stating that the person in front of the window is not Jesus is presenting reality, which indicates what is real without arguing or trying to convince the client. Agreeing with the delusion and asking the client what makes them think that Jesus is standing by the window would not address the inaccurate perception. Stating that it is just a delusion does not directly address the inaccurate perception of reality. Test-Taking Tip: Identify option components as correct or incorrect. This may help you identify a wrong answer. p. 99

The primary care provider has informed the pregnant client of several options for follow-up after the finding of an abnormality on the 5-month ultrasound, which includes genetic testing, amniocentesis, etc. Which approach by the nurse would be most useful? "What do you consider to be options you would be comfortable with?" "I would advise you to complete all of these as soon as possible for your peace of mind." "Amniocentesis could result in loss of the pregnancy; it would not be a risk I would be willing to take if I were you." "Here are some brochures which discuss all of the possible abnormalities and what they mean, please let me know if you have any questions."

"What do you consider to be options you would be comfortable with?" Asking the client what she considers to be possible options supports critical thinking on the part of the client as she attempts to find a solution. Offering advice such as telling her to complete the testing quickly or that she should not take the risk interferes with the client's decision-making process and is not therapeutic. Handing the client brochures implies the nurse does not want to talk about the client's concerns and can create a nontherapeutic barrier. p. 99

The nursing instructor is speaking with a recent nursing graduate who failed the national licensing exam on the first attempt. The instructor asks the student: "Why didn't you complete a review course like I suggested?" Which alternative question might be more therapeutic? "What strategies did you use to study for the exam?" "How angry are your parents that you failed on the first try?" "Do you really believe that you did all you could to pass the exam?" "Do you feel that the school did not prepare you well enough for the licensing exam?"

"What strategies did you use to study for the exam?" "Why" questions imply criticism. It is better to ask questions such as what strategies the student nurse used to focus more on facilitating problem solving. Asking how angry the students' parents are about the failure, whether the student believes that he or she prepared enough, or if the school prepared the student enough does not focus on problem solving and would not help to focus on solving the current problem. p. 96

Which comment by the nurse would be appropriate when beginning a new nurse-client relationship? "Which of your problems is most serious?" "I want you to tell me about your problems." "I'm an experienced nurse. You can trust me." "What would you like to tell me about yourself?"

"What would you like to tell me about yourself?" Asking the client what he or she wants to tell the nurse is an open-ended statement that gives the client control over the interview. It emphasizes the client-centered nature of the nurse-client relationship. The focus is on the client's ideas, experiences, and feelings. Evaluating the client's problems and goals comes later in the working phase of the nurse-client relationship. Bringing up problems right away is not an open-ended approach and can make the client uncomfortable. Bringing up the nurse's experience does not keep the focus on the client. p. 120

A nurse is interacting with a child during a home visit. The nurse learns that the child is depressed because of poor grades in school. When the nurse responds, the child says the nurse just doesn't understand. Which nontherapeutic response might the nurse have used to cause this reaction? "Why did you score low in your exams?" "What are you worried about?" "You must be feeling very upset." "What would you like to talk about?"

"Why did you score low in your exams?" The response, "Why did you score low on your exams?" is a nontherapeutic response because the nurse is probing a topic that is sensitive for the child. The nurse should strive to decrease the client's anxiety, and a question like this will likely increase it. The child may then feel that the nurse is unable to understand his or her feelings. The response, "What are you worried about?" is a therapeutic response. The child can then help the nurse understand and acknowledge his or her feelings. The response, "You must be feeling very upset" is a therapeutic response. The nurse has restated the child's feelings to seek clarification. The response, "What would you like to talk about?" is a therapeutic response. It is an example of broad opening in which the nurse encourages the child to share his or her feelings. p. 120

A nurse assesses a new client whose chief concern is "daily crying spells." Which comment from the client would prompt the nurse to suspect a medical reason is causing the problem rather than depression? "I usually drink two or three cups of coffee in the morning." "I often have headaches, especially when the pollen count is high." "Years ago I had thyroid problems, but they cleared up and I stopped the medicine." "I recently had three moles removed because my doctor thought they were suspicious."

"Years ago I had thyroid problems, but they cleared up and I stopped the medicine." Endocrine disorders like hypothyroidism may have the clinical appearance of depression, and the client has indicated that he or she no longer takes medication for the thyroid problems. Crying spells or symptoms of depression would not be linked to drinking coffee, allergies to pollen, or the appearance of moles. pp. 79-80

A pregnant client with a history of three successive miscarriages is worried and says, "I'm afraid I may miscarry again, and my husband would be disappointed." What is the most therapeutic response by the nurse? "I feel so sorry for you." "You shouldn't talk like that." "Don't be upset; it is all going to be fine." "You are worried about having a miscarriage."

"You are worried about having a miscarriage." Clients are encouraged to share their feelings. Therefore, restating that the client is worried about having a miscarriage is reflective listening that encourages the client to talk more. Feeling sorry for the client is a sympathetic response and limits the client's response. Telling the client not to do something would make her feel ignored. Telling the client that all will be fine may limit the conversation and is a false reassurance. p. 121

A nurse is interacting with a client who is diagnosed with human immunodeficiency virus (HIV). The nurse observes that the client appears depressed and avoids interacting whenever possible. What response should the nurse provide to encourage the client to acknowledge his or her feelings? "You look upset about something." "Don't worry about what others would say." "Are you thinking about your illness now?" "I feel you are not willing to interact with me."

"You look upset about something." Observing that the client looks upset invites a response while keeping the focus on the client's feelings. It makes the client feel that the nurse is concerned about him or her and encourages sharing feelings. The nurse should not be judgmental and come to a conclusion that the client is worried about what others would say. The nurse should avoid asking closed-ended questions such as, "Are you thinking about your illness?" It limits the conversation to a "yes" or "no" response and may interfere with further communication. It is unprofessional for the nurse to express his or her own feelings. p. 120

A nurse counsels a widow whose husband died five years ago. The widow says, "If I'd done more, he would still be alive." Which is the nurse's most therapeutic response? "I understand how you feel after such a terrible loss." "That was a long time ago. It's time to move on with your life." "You did a good job of caring for him. There was nothing else you could have done." "Your husband was 82 years old with severe chronic obstructive pulmonary disease."

"Your husband was 82 years old with severe chronic obstructive pulmonary disease." Pointing out that the husband was 82 and had severe chronic obstructive pulmonary disease demonstrates the therapeutic technique of presenting reality. The nurse is simply presenting the facts of the situation without judgment or comment. Giving advice, disagreeing, and changing the subject are nontherapeutic communication techniques. Though many people use them in everyday life, they can be problematic when applied to clients. p. 97

When caring for clients from which cultures would the nurse need to be aware that touch practices are infrequent and may be considered inappropriate or uncomfortable? Select all that apply. Japan Italian British American Chinese American Mexican American

-Japan -British American -Chinese American Patients from Japan, British Americans, and Chinese Americans may not like to be touched and use touch infrequently. This is less of a concern for Italian and Mexican American clients. p. 101

Which interventions are most appropriate for a basic level psychiatric mental health registered nurse (PMH-RN)? Managing the milieu by selecting activities for an adolescent group Conducting a couples psychotherapy group focusing on effective parenting Assisting a client's family in identifying appropriate housing for their parent Presenting information on the special needs of the depressed to a family support group

-Managing the milieu by selecting activities for an adolescent group -Assisting a client's family in identifying appropriate housing for their parent -Presenting information on the special needs of the depressed to a family support group The interventions that are appropriate for a PMH-RN to implement include milieu management with the selection of activities for an adolescent group, assisting a client's family in identifying appropriate housing for his or her parent, and presenting information on the special needs of depressed people to a family support group. Conducting a couples' psychotherapy group focusing on effective parenting is incorrect, because it is a prescriptive treatment that can only be performed by a psychiatric mental health advanced practice registered nurse. p. 86

Which activities completed by the nurse would support the process of active listening? Select all that apply. Including frequent periods of silence. Monitoring his or her own nonverbal responses. Observing the client's nonverbal behaviors. Learning to quiet oneself to avoid the urge to help. Providing the clients with feedback about themselves of which they may be unaware.

-Monitoring his or her own nonverbal responses. -Observing the client's nonverbal behaviors. -Learning to quiet oneself to avoid the urge to help. -Providing the clients with feedback about themselves of which they may be unaware. Active listening requires the nurse to be available psychologically, socially and emotionally. Monitoring one's own nonverbal responses, observing the client's nonverbal behaviors, learning to quiet oneself to avoid the urge to help, and providing the clients with feedback about themselves of which they may be unaware are all activities that support active listening. Silence can be useful during interactions and its use is determined by each specific situation. Using silence frequently does not necessarily support active listening. p. 95

Which statements are true about mental illness in children? Regression is a hallmark of psychiatric disorders in children. Interviewing a child without a caregiver present is prohibited. An estimated one in 20 children in the United States suffer from a mental illness. Observing a child at play is an effective means of assessing psychiatric well-being. Parents and guardians are the best source for determining a child's inner feelings.

-Regression is a hallmark of psychiatric disorders in children. -Observing a child at play is an effective means of assessing psychiatric well-being. Regressing to a previous stage of development is a hallmark of psychiatric disorders in children. For example, thumb sucking is normal for toddlers but not older children. Observing a child at play is an effective means of assessing psychiatric well-being and should be combined with the clinical interview. Interviewing a child without a caregiver present may be appropriate if a child is reluctant to share information, especially in cases of suspected abuse. An estimated 1 in 10 children in the United States suffers from a mental illness. Parents and guardians may provide important information, but the child is the best source in determining his or her own inner feelings. pp. 77-78

Which are potential advantages of telehealth for health care? Select all that apply. Relieve the impending nursing shortage. Provide opportunities for virtual health examinations. Eliminate the need for primary care provider involvement. Allow clients to virtually manage their own health care. Eliminate potential errors as people are replaced by technology.

-Relieve the impending nursing shortage. -Allow clients to virtually manage their own health care. -Provide opportunities for virtual health examinations. Telehealth has the potential to relieve the impending nursing shortage, provide virtual access to clients, and allow clients to virtually manage their own health care. Telehealth will not eliminate the need for primary care provider involvement, and although errors can be reduced through the use of technology, technology alone cannot eliminate all errors. Test-Taking Tip: Be alert for details about what you are being asked to do. In this question type, you are asked to select all options that apply to a given situation or patient. All options likely relate to the situation, but only some of the options may relate directly to the situation. p. 101

Which of the following patients may be an appropriate candidate for a release from hospitalization known as against medical advice (AMA)? a. 37-year-old patient hospitalized for 6 days; the provider feels one more day would benefit the patient, but the patient doesn't agree and wishes to be discharged b. 75-year-old patient with dementia who demands to be allowed to go back to his own home c. 21-year-old actively suicidal patient on the psychiatric unit who wants to be discharged to home and do outpatient counseling d. 32-year-old female patient who wishes to stay in the hospital but whose husband demands that she be discharged into his care

A Against medical advice discharges are sometimes used when the patient does not agree with the provider, as long as the patient is not a danger to himself or herself or to others. The patient with dementia and the patient who is actively suicidal would pose a safety risk and would be not allowed to be discharged AMA. A patient who wishes to stay in the hospital can make that decision; a family member's opinion doesn't impact an AMA discharge. Cognitive Level: Analyze (Analysis) Nursing Process: Implementation NCLEX: Safe and Effective Care Environment Text page: 101

A client is released from involuntary commitment by the judge, who orders that a caseworker supervise him for the next 6 months. This is an example of a. conditional discharge. b. outpatient commitment. c. voluntary follow-up. d. discretionary treatment.

A An unconditional discharge gives the client complete freedom to choose or reject follow-up care. A conditional discharge imposes a legal requirement for the client to submit to follow-up supervision. REF: Page 101

The use of seclusion or restraint to control the behavior of a client who is at risk of harming self or others gives rise to conflict between the ethical principles of a. autonomy and beneficence. b. advocacy and confidentiality. c. veracity and fidelity. d. justice and humanism.

A Autonomy refers to self-determination and beneficence refers to doing good. When a client is restrained or secluded, the need to do good and prevent harm outweighs the client's autonomy. REF: Page 99

A nurse is adequately representing the stated bioethical principle when valuing a. autonomy by respecting a client's right to decide to refuse cancer treatment. b. justice by staying with a client who is suicidal. c. fidelity by informing the client about the negative side effects of a proposed treatment. d. beneficence when advocating for a client's right to enter into a clinical trial for a new medication.

A Autonomy refers to self-determination. Self-determination can be exercised when one makes his or her own decisions without interference from others. REF: Page 99

The charge nurse shares with the psychiatric technician that negligence of a patient a. is an act or failure to act in a way that a responsible employee would act. b. applies only when the client is abandoned or mistreated. c. is an action that puts the client in fear of being harmed by the employee. d. means the employee has given malicious false information about the client.

A Behaving as a wise and prudent person would act under the same circumstances is one way of judging whether the standard of care has been violated. Employers typically hope that staff will prevent clients from striking each other. REF: Page 107-108

Jonas is a 29-year-old patient with anxiety and a history of alcohol abuse who is an inpatient on the psychiatric unit. He becomes angry and aggressive, strikes another patient, and then attacks a staff member. He is taken to seclusion and medicated with haloperidol and lorazepam. In this case, the haloperidol and lorazepam may be considered: a. a restraint. b. a medication time-out. c. false imprisonment. d. malpractice.

A Chemical restraints are defined by those medications or doses of medication that are not being used for the patient's condition. Medication time-out is incorrect; false imprisonment and malpractice refer to specific legal terms that do not have any bearing on this medication scenario. Cognitive Level: Apply (Application) Nursing Process: Implementation NCLEX: Safe and Effective Care Environment Text page: 104

The civil rights of persons with mental illness who are hospitalized for treatment are a. the same as those for any other citizen. b. altered to prevent use of poor judgment. c. always ensured by appointment of a guardian. d. limited to provision of humane treatment.

A Civil rights are not lost because of hospitalization for mental illness. REF: 99-100

In the course of providing best psychiatric care for a client, the nurse must place greatest reliance on a. legal principles. b. ethical principles. c. independent judgment. d. institutional standards.

A Legal principles are fundamental to nursing practice. They supersede all other principles, standards, and judgments. All students are encouraged to become familiar with the important provisions of the laws in their own states regarding admissions, discharges, clients' rights, and informed consent. REF: 99-100

Which statement concerning the right to treatment in public psychiatric hospitals is accurate? a. Hospitalization without treatment violates the client's rights. b. Right to treatment extends only to provision of food, shelter, and safety. c. All clients have the right to choose a primary therapist and case manager. d. The right to treatment for hallucinations has priority over treatment for anxiety.

A Many years ago psychiatric clients were warehoused in large mental institutions, given custodial care, and rarely released into the community. As enlightenment occurred, it was determined that each client who is hospitalized has the right to receive treatment. REF: Page 101-102

Gina is admitted for treatment of depression with suicidal ideation triggered by marital discord. Her spouse visits one night and informs Gina that he has decided to file for divorce. The staff are aware of the visit and the husband's intentions regarding divorce, but take no further action, feeling that the q15-minute suicide checks Gina is already on are sufficient. Thirty minutes after the visit ends, staff make rounds and discover Gina has hanged herself in the bathroom, using hospital pajamas she has tied together into a rope. Which of the following statements best describes the situation. SELECT ALL THAT APPLY a. The nurses have created liability for themselves and their employer by failing in their duty to protect Gina. b. The nurses have breached their duty to reassess Gina for increased suicide risk after her husband's visit. c. Given Gina's history, the nurses should have expected an increased risk of suicide after the husband's announcement. d. The nurses correctly reasoned that suicides cannot always be prevented and did their best to keep Gina safe through checks every fifteen minutes. e. The nurses are subject to a tort of professional negligence for failing to prevent the suicide by increasing the suicide precautions in response to Gina's increased risk. f. Had the nurses restricted Gina's movements or increased their checks on her, the would have been liable for false imprisonment and invasion of privacy, respectfully.

A, B, C, E

A patient diagnosed with major depression has lost 20 pounds in one month, has chronic low self-esteem, and a plan for suicide. The patient has taken an antidepressant medication for 1 week. Which nursing intervention has the highest priority? a. Implement suicide precautions. b. Offer high-calorie snacks and fluids frequently. c. Assist the patient to identify three personal strengths. d. Observe patient for therapeutic effects of antidepressant medication.

ANS: A Implementing suicide precautions is the only option related to patient safety. The other options, related to nutrition, self-esteem, and medication therapy, are important but are not priorities.

When a nurse assesses an older adult patient, answers seem vague or unrelated to the questions. The patient also leans forward and frowns, listening intently to the nurse. An appropriate question for the nurse to ask would be: a. "Are you having difficulty hearing when I speak?" b. "How can I make this assessment interview easier for you?" c. "I notice you are frowning. Are you feeling annoyed with me?" d. "You're having trouble focusing on what I'm saying. What is distracting you?"

ANS: A The patient's behaviors may indicate difficulty hearing. Identifying any physical need the patient may have at the onset of the interview and making accommodations are important considerations. By asking if the patient is annoyed, the nurse is jumping to conclusions. Asking how to make the interview easier for the patient may not elicit a concrete answer. Asking about distractions is a way of asking about auditory hallucinations, which is not appropriate because the nurse has observed that the patient seems to be listening intently.

*A patient says, "Please don't share information about me with the other people." How should the nurse respond?* a. "I will not share information with your family or friends without your permission, but I share information about you with other staff." b. "A therapeutic relationship is just between the nurse and the patient. It is up to you to tell others what you want them to know." c. "It depends on what you choose to tell me. I will be glad to disclose at the end of each session what I will report to others." d. "I cannot tell anyone about you. It will be as though I am talking about my own problems, and we can help each other by keeping it between us."

ANS: A A patient has the right to know with whom the nurse will share information and that confidentiality will be protected. Although the relationship is primarily between the nurse and patient, other staff needs to know pertinent data. The other incorrect responses promote incomplete disclosure on the part of the patient, require daily renegotiation of an issue that should be resolved as the nurse-patient contract is established, and suggest mutual problem solving. The relationship must be patient centered.

A community mental health nurse has worked with a patient for 3 years but is moving out of the city and terminates the relationship. When a novice nurse begins work with this patient, what is the starting point for the relationship? a. Begin at the orientation phase. b. Resume the working relationship. c. Initially establish a social relationship. d. Return to the emotional catharsis phase.

ANS: A After termination of a long-term relationship, the patient and new nurse usually have to begin at ground zero, the orientation phase, to build a new relationship. If termination is successfully completed, the orientation phase sometimes progresses quickly to the working phase. Other times, even after successful termination, the orientation phase may be prolonged.

A patient says, "I've done a lot of cheating and manipulating in my relationships." Select a nonjudgmental response by the nurse. a. "How do you feel about that?" b. "I am glad that you realize this." c. "That's not a good way to behave." d. "Have you outgrown that type of behavior?"

ANS: A Asking a patient to reflect on feelings about his or her actions does not imply any judgment about those actions, and it encourages the patient to explore feelings and values. The remaining options offer negative judgments.

11. A Mexican-American patient puts a picture of the Virgin Mary on the bedside table. Under which section of the assessment should the nurse document this behavior? a. Culture b. Ethnicity c. Verbal communication d. Nonverbal communication

ANS: A Cultural heritage is expressed through language, works of art, music, dance, ethnic clothing, customs, traditions, diet, and expressions of spirituality. This patient's prominent placement of the picture is an example of expression of cultural heritage.

Which statement shows a nurse has empathy for a patient who made a suicide attempt? a. "You must have been very upset when you tried to hurt yourself." b. "It makes me sad to see you going through such a difficult experience." c. "If you tell me what is troubling you, I can help you solve your problems." d. "Suicide is a drastic solution to a problem that may not be such a serious matter."

ANS: A Empathy permits the nurse to see an event from the patient's perspective, understand the patient's feelings, and communicate this to the patient. The incorrect responses are nurse- centered (focusing on the nurse's feelings rather than the patient's), belittling, and sympathetic.

9. While talking with a patient with severe depression, a nurse notices the patient is unable to maintain eye contact. The patient's chin lowers to the chest while the patient looks at the floor. Which aspect of communication has the nurse assessed? a. Nonverbal communication b. A message filter c. A cultural barrier d. Social skills

ANS: A Eye contact and body movements are considered nonverbal communication. Insufficient data are available to determine the level of the patient's social skills or whether a cultural barrier exists.

18. A patient is having difficulty making a decision. The nurse has mixed feelings about whether to provide advice. Which principle usually applies? Giving advice: a. Is rarely helpful. b. Fosters independence. c. Lifts the burden of personal decision making. d. Helps the patient develop feelings of personal adequacy.

ANS: A Giving advice fosters dependence on the nurse and interferes with the patient's right to make personal decisions. Giving advice also robs patients of the opportunity to weigh alternatives and to develop problem-solving skills. Furthermore, it contributes to patient feelings of personal inadequacy. It also keeps the nurse in control and feeling powerful.

A nurse explains to the family of a mentally ill patient how a nurse-patient relationship differs from social relationships. Which is the best explanation? a. "The focus is on the patient. Problems are discussed by the nurse and patient, but solutions are implemented by the patient." b. "The focus shifts from nurse to patient as the relationship develops. Advice is given by both, and solutions are implemented." c. "The focus of the relationship is socialization. Mutual needs are met, and feelings are shared openly." d. "The focus is creation of a partnership in which each member is concerned with growth and satisfaction of the other."

ANS: A Only the correct response describes elements of a therapeutic relationship. The remaining responses describe events that occur in social or intimate relationships.

5. Which technique will best communicate to a patient that the nurse is interested in listening? a. Restate a feeling or thought the patient has expressed. b. Ask a direct question, such as "Did you feel angry?" c. Make a judgment about the patient's problem. d. Say, "I understand what you're saying."

ANS: A Restating allows the patient to validate the nurse's understanding of what has been communicated. Restating is an active listening technique. Judgments should be suspended in a nurse-patient relationship. Closed-ended questions such as, "Did you feel angry?" ask for specific information rather than show understanding. When the nurse simply states that he or she understands the patient's words, the patient has no way of measuring the understanding.

14. When a female Mexican-American patient and a female nurse sit together, the patient often holds the nurse's hand. The patient also links arm and arm with the nurse when they walk. The nurse is uncomfortable with this behavior and thinks the patient is homosexual. Which alternative is a more accurate assessment? a. The patient is accustomed to touch during conversations, as are members of many Hispanic subcultures. b. The patient understands that touch makes the nurse uncomfortable and controls the relationship based on that factor. c. The patient is afraid of being alone. When touching the nurse, the patient is reassured and comforted. d. The nurse is homophobic.

ANS: A The most likely answer is that the patient's behavior is culturally influenced. Hispanic women frequently touch women they consider to be their friends. Although the other options are possible, they are less likely.

As a nurse discharges a patient, the patient gives the nurse a card of appreciation made in an arts and crafts group. What is the nurse's best action? a. Recognize the effectiveness of the relationship and patient's thoughtfulness. Accept the card. b. Inform the patient that accepting gifts violates policies of the facility. Decline the card. c. Acknowledge the patient's transition through the termination phase but decline the card. d. Accept the card and invite the patient to return to participate in other arts and crafts groups.

ANS: A The nurse must consider the meaning, timing, and value of the gift. In this instance, the nurse should accept the patient's expression of gratitude.

At what point in the nurse-patient relationship should a nurse plan to first address termination? a. During the orientation phase b. At the end of the working phase c. Near the beginning of the termination phase d. When the patient initially brings up the topic

ANS: A The patient has a right to know the conditions of the nurse-patient relationship. If the relationship is to be time-limited, the patient should be informed of the number of sessions. If it is open-ended, the termination date will not be known at the outset, and the patient should know that the issue will be negotiated at a later date. The nurse is responsible for bringing up the topic of termination early in the relationship, usually during the orientation phase.

A nurse says, "I am the only one who truly understands this patient. Other staff members are too critical." The nurse's statement indicates: a. boundary blurring. c. positive regard. b. sexual harassment. d. advocacy.

ANS: A When the role of the nurse and the role of the patient shift, boundary blurring may arise. In this situation the nurse is becoming over-involved with the patient as a probable result of unrecognized countertransference. When boundary issues occur, the need for supervision exists. The situation does not describe sexual harassment. Data are not present to suggest positive regard or advocacy.

A nurse ends a relationship with a patient. Which actions by the nurse should be included in the termination phase? Select all that apply. a. Focus dialogues with the patient on problems that may occur in the future. b. Help the patient express feelings about the relationship with the nurse. c. Help the patient prioritize and modify socially unacceptable behaviors. d. Reinforce expectations regarding the parameters of the relationship. e. Help the patient to identify strengths, limitations, and problems.

ANS: A, B The correct actions are part of the termination phase. The other actions would be used in the working and orientation phases.

2. Which benefits are most associated with the use of telehealth? Select all that apply. a. Cost savings for patients b. Maximization of care management c. Access to services for patients in rural areas d. Prompt reimbursement by third-party payers e. Rapid development of trusting relationships with patients

ANS: A, B, C Use of telehealth technologies has shown that it can maximize health and improve disease management skills and confidence with the disease process. Many rural patients have felt disconnected from services; telehealth technologies can solve these problems. Although telehealth's improved health outcomes regularly show cost savings for payers, one significant barrier is the current lack of reimbursement for remote patient monitoring by third-party payers. Telehealth is not associated with rapid development of trusting relationships.

A nurse performing an assessment interview for a patient with a substance use disorder decides to use a standardized rating scale. Which scales are appropriate? Select all that apply. a. Addiction Severity Index (ASI) b. Brief Drug Abuse Screen Test (B-DAST) c. Abnormal Involuntary Movement Scale (AIMS) d. Cognitive Capacity Screening Examination (CCSE) e. Recovery Attitude and Treatment Evaluator (RAATE)

ANS: A, B, E Standardized scales are useful for obtaining data about substance use disorders. The ASI, B-DAST, and RAATE are scales related to substance abuse. AIMS assesses involuntary movements associated with anti-psychotic medications. The CCSE assesses cognitive function.

A novice psychiatric nurse has a parent with bipolar disorder. This nurse angrily recalls feelings of embarrassment about the parent's behavior in the community. Select the best ways for this nurse to cope with these feelings. Select all that apply. a. Seek ways to use the understanding gained from childhood to help patients cope with their own illnesses. b. Recognize that these feelings are unhealthy. The nurse should try to suppress them when working with patients. c. Recognize that psychiatric nursing is not an appropriate career choice. Explore other nursing specialties. d. The nurse should begin new patient relationships by saying, "My own parent had mental illness, so I accept it without stigma." e. Recognize that the feelings may add sensitivity to the nurse's practice, but supervision is important.

ANS: A, E The nurse needs support to explore these feelings. An experienced psychiatric nurse is a resource that may be helpful. The knowledge and experience gained from the nurse's relationship with a mentally ill parent may contribute sensitivity to compassionate practice. Self-disclosure and suppression are not adaptive coping strategies. The nurse should not give up on this area of practice without first seeking ways to cope with the memories.

A nurse asks a patient, "If you had fever and vomiting for 3 days, what would you do?" Which aspect of the mental status examination is the nurse assessing? a. Behavior b. Cognition c. Affect and mood d. Perceptual disturbances

ANS: B Assessing cognition involves determining a patient's judgment and decision making. In this case, the nurse would expect a response of "Call my doctor" if the patient's cognition and judgment are intact. If the patient responds, "I would stop eating" or "I would just wait and see what happened," the nurse would conclude that judgment is impaired. The other options refer to other aspects of the examination.

Before assessing a new patient, a nurse is told by another health care worker, "I know that patient. No matter how hard we work, there isn't much improvement by the time of discharge." The nurse's responsibility is to: a. document the other worker's assessment of the patient. b. assess the patient based on data collected from all sources. c. validate the worker's impression by contacting the patient's significant other. d. discuss the worker's impression with the patient during the assessment interview.

ANS: B Assessment should include data obtained from both the primary and reliable secondary sources. The nurse, bearing in mind the possible effects of counter-transference, should evaluate biased assessments by others as objectively as possible.

A patient presents to the emergency department with mixed psychiatric symptoms. The admission nurse suspects the symptoms may be the result of a medical problem. Lab results show elevated BUN (blood urea nitrogen) and creatinine. What is the nurse's next best action? a. Report the findings to the health care provider. b. Assess the patient for a history of renal problems. c. Assess the patient's family history for cardiac problems. d. Arrange for the patient's hospitalization on the psychiatric unit.

ANS: B Elevated BUN (blood urea nitrogen) and creatinine suggest renal problems. Renal dysfunction can often imitate psychiatric disorders. The nurse should further assess the patient's history for renal problems and then share the findings with the health care provider.

A new staff nurse completes an orientation to the psychiatric unit. This nurse will expect to ask an advanced practice nurse to perform which action for patients? a. Perform mental health assessment interviews. b. Prescribe psychotropic medication. c. Establish therapeutic relationships. d. Individualize nursing care plans.

ANS: B Prescriptive privileges are granted to master's-prepared nurse practitioners who have taken special courses on prescribing medication. The nurse prepared at the basic level performs mental health assessments, establishes relationships, and provides individualized care planning. Note that this question was also offered for Chapter 1.

Which entry in the medical record best meets the requirement for problem-oriented charting? a. "A: Pacing and muttering to self. P: Sensory perceptual alteration related to internal auditory stimulation. I: Given fluphenazine HCL (Prolixin) 2.5 mg po at 0900 and went to room to lie down. E: Calmer by 0930. Returned to lounge to watch TV." b. "S: States, 'I feel like I'm ready to blow up.' O: Pacing hall, mumbling to self. A: Auditory hallucinations. P: Offer haloperidol (Haldol) 2 mg po. I: Haloperidol (Haldol) 2 mg po given at 0900. E: Returned to lounge at 0930 and quietly watched TV." c. "Agitated behavior. D: Patient muttering to self as though answering an unseen person. A: Given haloperidol (Haldol) 2 mg po and went to room to lie down. E: Patient calmer. Returned to lounge to watch TV." d. "Pacing hall and muttering to self as though answering an unseen person. haloperidol (Haldol) 2 mg po administered at 0900 with calming effect in 30 minutes. Stated, 'I'm no longer bothered by the voices.'"

ANS: B Problem-oriented documentation uses the first letter of key words to organize data: S for subjective data, O for objective data, A for assessment, P for plan, I for intervention, and E for evaluation. The distracters offer examples of PIE charting, focus documentation, and narrative documentation.

"QSEN" refers to: a. Qualitative Standardized Excellence in Nursing b. Quality and Safety Education for Nurses c. Quantitative Effectiveness in Nursing d. Quick Standards Essential for Nurses

ANS: B QSEN represents national initiatives centered on patient safety and quality. The primary goal of QSEN is to prepare future nurses with the knowledge, skills, and attitudes to increase the quality, care, and safety in the health care setting in which they work.

A nurse wants to assess an adult patient's recent memory. Which question would best yield the desired information? a. "Where did you go to elementary school?" b. "What did you have for breakfast this morning?" c. "Can you name the current president of the United States?" d. "A few minutes ago, I told you my name. Can you remember it?"

ANS: B The patient's recall of a meal provides evidence of recent memory. Two incorrect responses are useful to assess immediate and remote memory. The other distracter assesses the patient's fund of knowledge.

After formulating the nursing diagnoses for a new patient, what is a nurse's next action? a. Designing interventions to include in the plan of care b. Determining the goals and outcome criteria c. Implementing the nursing plan of care d. Completing the spiritual assessment

ANS: B The third step of the nursing process is planning and outcome identification. Outcomes cannot be determined until the nursing assessment is complete and nursing diagnoses have been formulated.

A nurse introduces the matter of a contract during the first session with a new patient because contracts: a. specify what the nurse will do for the patient. b. spell out the participation and responsibilities of each party. c. indicate the feeling tone established between the participants. d. are binding and prevent either party from prematurely ending the relationship.

ANS: B A contract emphasizes that the nurse works with the patient rather than doing something for the patient. "Working with" is a process that suggests each party is expected to participate and share responsibility for outcomes. Contracts do not, however, stipulate roles or feeling tone, and premature termination is forbidden.

A nurse assesses a confused older adult. The nurse experiences sadness and reflects, "The patient is like one of my grandparents...so helpless." Which response is the nurse demonstrating? a. Transference b. Countertransference c. Catastrophic reaction d. Defensive coping reaction

ANS: B Countertransference is the nurse's transference or response to a patient that is based on the nurse's unconscious needs, conflicts, problems, or view of the world.

An advanced practice nurse observes a novice nurse expressing irritability regarding a patient with a long history of alcoholism and suspects the new nurse is experiencing countertransference. Which comment by the new nurse confirms this suspicion? a. "This patient continues to deny problems resulting from drinking." b. "My parents were alcoholics and often neglected our family." c. "The patient cannot identify any goals for improvement." d. "The patient said I have many traits like her mother."

ANS: B Countertransference occurs when the nurse unconsciously and inappropriately displaces onto the patient feelings and behaviors related to significant figures in the nurse's past. In this instance, the new nurse's irritability stems from relationships with parents. The distracters indicate transference or accurate analysis of the patient's behavior.

A nurse wants to demonstrate genuineness with a patient diagnosed with schizophrenia. The nurse should: a. restate what the patient says. b. use congruent communication strategies. c. use self-revelation in patient interactions. d. consistently interpret the patient's behaviors.

ANS: B Genuineness is a desirable characteristic involving awareness of one's own feelings as they arise and the ability to communicate them when appropriate. The incorrect options are undesirable in a therapeutic relationship.

12. An African-American patient says to a Caucasian nurse, "There's no sense talking. You wouldn't understand because you live in a white world." The nurse's best action would be to: a. explain, "Yes, I do understand. Everyone goes through the same experiences." b. say, "Please give an example of something you think I wouldn't understand." c. reassure the patient that nurses are in contact with people from all cultures. d. change the subject to one that is less emotionally disturbing.

ANS: B Having the patient speak in specifics rather than globally helps the nurse understand the patient's perspective. This approach helps the nurse engage the patient.

2. A patient with paranoid schizophrenia tells the nurse, "The CIA is monitoring us through the fluorescent lights in this room. Be careful what you say." Which response by the nurse would be most therapeutic? a. "Let's talk about something other than the CIA." b. "It sounds like you're concerned about your privacy." c. "The CIA is prohibited from operating in health care facilities." d. "You have lost touch with reality, which is a symptom of your illness."

ANS: B It is important not to challenge the patient's beliefs, even if they are unrealistic. Challenging undermines the patient's trust in the nurse. The nurse should try to understand the underlying feelings or thoughts the patient's message conveys. The correct response uses the therapeutic technique of reflection. The other comments are nontherapeutic. Asking to talk about something other than the concern at hand is changing the subject. Saying that the CIA is prohibited from operating in health care facilities gives false reassurance. Stating that the patient has lost touch with reality is truthful but uncompassionate.

3. The patient says, "My marriage is just great. My spouse and I usually agree on everything." The nurse observes the patient's foot moving continuously as the patient twirls a shirt button. What conclusion can the nurse draw? The patient's communication is: a. clear. b. mixed. c. precise. d. inadequate.

ANS: B Mixed messages involve the transmission of conflicting or incongruent messages by the speaker. The patient's verbal message that all is well in the relationship is modified by the nonverbal behaviors denoting anxiety. Data are not present to support the choice of the verbal message being clear, explicit, or inadequate.

What is the desirable outcome for the orientation stage of a nurse-patient relationship? The patient will demonstrate behaviors that indicate: a. self-responsibility and autonomy. b. a greater sense of independence. c. rapport and trust with the nurse. d. resolved transference.

ANS: C Development of rapport and trust is necessary before the relationship can progress to the working phase. Behaviors indicating a greater sense of independence, self-responsibility, and resolved transference occur in the working phase.

A nurse wants to enhance growth of a patient by showing positive regard. The nurse's action most likely to achieve this goal is: a. making rounds daily. b. staying with a tearful patient. c. administering medication as prescribed. d. examining personal feelings about a patient.

ANS: B Staying with a crying patient offers support and shows positive regard. Administering daily medication and making rounds are tasks that could be part of an assignment and do not necessarily reflect positive regard. Examining feelings regarding a patient addresses the nurse's ability to be therapeutic.

As a patient diagnosed with a mental illness is being discharged from a facility, a nurse invites the patient to the annual staff picnic. What is the best analysis of this scenario? a. The invitation facilitates dependency on the nurse. b. The nurse's action blurs the boundaries of the therapeutic relationship. c. The invitation is therapeutic for the patient's diversional activity deficit. d. The nurse's action assists the patient's integration into community living.

ANS: B The invitation creates a social relationship rather than a therapeutic relationship.

20. A patient with acute depression states, "God is punishing me for my past sins." What is the nurse's best response? a. "Why do you think that?" b. "You sound very upset about this." c. "You believe God is punishing you for your sins?" d. "If you feel this way, you should talk to a member of your clergy."

ANS: B The nurse reflects on the patient's comment, a therapeutic technique to encourage sharing for perceptions and feelings. The incorrect responses reflect probing, closed-ended comments, and giving advice, all of which are nontherapeutic.

10. During the first interview with a parent whose child died in a car accident, the nurse feels empathic and reaches out to take the patient's hand. Select the correct analysis of the nurse's behavior. a. It shows empathy and compassion. It will encourage the patient to continue to express feelings. b. The gesture is premature. The patient's cultural and individual interpretation of touch is unknown. c. The patient will perceive the gesture as intrusive and overstepping boundaries. d. The action is inappropriate. Patients in a psychiatric setting should not be touched.

ANS: B Touch has various cultural and individual interpretations. Nurses should refrain from using touch until an assessment can be made regarding the way in which the patient will perceive touch. The other options present prematurely drawn conclusions.

19. The relationship between a nurse and patient as it relates to status and power is best described by which term? a. Symmetric b. Complementary c. Incongruent d. Paralinguistic

ANS: B When a difference in power exists, as between a student and teacher or between a nurse and patient, the relationship is said to be complementary. Symmetrical relationships exist between individuals of like or equal status. Incongruent and paralinguistic are not terms used to describe relationships.

Which descriptors exemplify consistency regarding nurse-patient relationships? Select all that apply. a. Encouraging a patient to share initial impressions of staff b. Having the same nurse care for a patient on a daily basis c. Providing a schedule of daily activities to a patient d. Setting a time for regular sessions with a patient e. Offering solutions to a patient's problems

ANS: B, C, D Consistency implies predictability. Having the same nurse see the patient daily and provide a daily schedule of patient activities and a set time for regular sessions will help a patient predict what will happen during each day and develop a greater degree of security and comfort. Encouraging a patient to share initial impressions of staff and giving advice are not related to consistency and would not be considered a therapeutic intervention.

1. A patient cries as the nurse explores the patient's relationship with a deceased parent. The patient says, "I shouldn't be crying like this. It happened a long time ago." Which responses by the nurse will facilitate communication? Select all that apply. a. "Why do you think you are so upset?" b. "I can see that you feel sad about this situation." c. "The loss of your parent is very painful for you." d. "Crying is a way of expressing the hurt you're experiencing." e. "Let's talk about something else because this subject is upsetting you.

ANS: B, C, D Reflecting ("I can see that you feel sad" or "This is very painful for you") and giving information ("Crying is a way of expressing hurt") are therapeutic techniques. "Why" questions often imply criticism or seem intrusive or judgmental, and they are difficult to answer. Changing the subject is a barrier to communication.

A patient is very suspicious and states, "The FBI has me under surveillance." Which strategies should a nurse use when gathering initial assessment data about this patient? Select all that apply. a. Tell the patient that medication will help this type of thinking. b. Ask the patient, "Tell me about the problem as you see it." c. Seek information about when the problem began. d. Tell the patient, "Your ideas are not realistic." e. Reassure the patient, "You are safe here."

ANS: B, C, E During the assessment interview, the nurse should listen attentively and accept the patient's statements in a nonjudgmental way. Because the patient is suspicious and fearful, reassuring safety may be helpful, although trust is unlikely so early in the relationship. Saying that medication will help or telling the patient that the ideas are not realistic will undermine development of trust between the nurse and patient.

1. A nurse assessed a patient who reluctantly participated in activities, answered questions with minimal responses, and rarely made eye contact. What information should be included when documenting the assessment? Select all that apply. a. The patient was uncooperative b. The patient's subjective responses c. Only data obtained from the patient's verbal responses d. A description of the patient's behavior during the interview e. Analysis of why the patient was unresponsive during the interview

ANS: B, D Both content and process of the interview should be documented. Providing only the patient's verbal responses would create a skewed picture of the patient. Writing that the patient was uncooperative is subjectively worded. An objective description of patient behavior would be preferable. Analysis of the reasons for the patient's behavior would be speculation, which is inappropriate.

What information is conveyed by nursing diagnoses? Select all that apply. a. Medical judgments about the disorder b. Unmet patient needs currently present c. Goals and outcomes for the plan of care d. Supporting data that validate the diagnoses e. Probable causes that will be targets for nursing interventions

ANS: B, D, E Nursing diagnoses focus on phenomena of concern to nurses rather than on medical diagnoses.

A novice nurse tells a mentor, "I want to convey to my patients that I am interested in them and that I want to listen to what they have to say." Which behaviors will be helpful in meeting the nurse's goal? Select all that apply. a. Sitting behind a desk, facing the patient b. Introducing self to a patient and identifying own role c. Maintaining control of discussions by asking direct questions d. Using facial expressions to convey interest and encouragement e. Assuming an open body posture and sometimes mirror imaging

ANS: B, D, E Trust is fostered when the nurse gives an introduction and identifies his or her role. Facial expressions that convey interest and encouragement support the nurse's verbal statements to that effect and strengthen the message. An open body posture conveys openness to listening to what the patient has to say. Mirror imaging enhances patient comfort. A desk would place a physical barrier between the nurse and patient. A face-to-face stance should be avoided when possible and a less intense 90- or 120-degree angle used to permit either party to look away without discomfort.

An adolescent asks a nurse conducting an assessment interview, "Why should I tell you anything? You'll just tell my parents whatever you find out." Which response by the nurse is appropriate? a. "That isn't true. What you tell us is private and held in strict confidence. Your parents have no right to know." b. "Yes, your parents may find out what you say, but it is important that they know about your problems." c. "What you say about feelings is private, but some things, like suicidal thinking, must be reported to the treatment team." d. "It sounds as though you are not really ready to work on your problems and make changes."

ANS: C Adolescents are very concerned with confidentiality. The patient has a right to know that most information will be held in confidence but that certain material must be reported or shared with the treatment team, such as threats of suicide, homicide, use of illegal drugs, or issues of abuse. The incorrect responses are not true, will not inspire the confidence of the patient, or are confrontational.

A patient begins a new program to assist with building social skills. In which part of the plan of care should a nurse record the item, "Encourage patient to attend one psychoeducational group daily"? a. Assessment b. Analysis c. Implementation d. Evaluation

ANS: C Interventions are the nursing prescriptions to achieve the outcomes. Interventions should be specific.

When a new patient is hospitalized, a nurse takes the patient on a tour, explains rules of the unit, and discusses the daily schedule. The nurse is engaged in: a. counseling. b. health teaching. c. milieu management. d. psychobiological intervention.

ANS: C Milieu management provides a therapeutic environment in which the patient can feel comfortable and safe while engaging in activities that meet the patient's physical and mental health needs. Counseling refers to activities designed to promote problem solving and enhanced coping and includes interviewing, crisis intervention, stress management, and conflict resolution. Health teaching involves identifying health education needs and giving information about these needs. Psychobiological interventions involve medication administration and monitoring response to medications.

Select the most appropriate label to complete this nursing diagnosis: ___________ related to feelings of shyness and poorly developed social skills as evidenced by watching television alone at home every evening. a. Deficient knowledge b. Ineffective coping c. Social isolation d. Powerlessness

ANS: C Nursing diagnoses are selected based on the etiological factors and assessment findings, or evidence. In this instance, the evidence shows social isolation that is caused by shyness and poorly developed social skills.

A newly admitted patient diagnosed with major depression has gained 20 pounds over a few months and has suicidal ideation. The patient has taken an antidepressant medication for 1 week without remission of symptoms. Select the priority nursing diagnosis. a. Imbalanced nutrition: more than body requirements b. Chronic low self-esteem c. Risk for suicide d. Hopelessness

ANS: C Risk for suicide is the priority diagnosis when the patient has both suicidal ideation and a plan to carry out the suicidal intent. Imbalanced nutrition, hopelessness, and chronic low self-esteem may be applicable nursing diagnoses, but these problems do not affect patient safety as urgently as would a suicide attempt.

Which behavior shows that a nurse values autonomy? The nurse: a. suggests one-on-one supervision for a patient who has suicidal thoughts. b. informs a patient that the spouse will not be in during visiting hours. c. discusses options and helps the patient weigh the consequences. d. sets limits on a patient's romantic overtures toward the nurse.

ANS: C A high level of valuing is acting on one's belief. Autonomy is supported when the nurse helps a patient weigh alternatives and their consequences before the patient makes a decision. Autonomy or self-determination is not the issue in any of the other behaviors.

As a nurse escorts a patient being discharged after treatment for major depression, the patient gives the nurse a necklace with a heart pendant and says, "Thank you for helping mend my broken heart." Which is the nurse's best response? a. "Accepting gifts violates the policies and procedures of the facility." b. "I'm glad you feel so much better now. Thank you for the beautiful necklace." c. "I'm glad I could help you, but I can't accept the gift. My reward is seeing you with a renewed sense of hope." d. "Helping people is what nursing is all about. It's rewarding to me when patients recognize how hard we work."

ANS: C Accepting a gift creates a social rather than therapeutic relationship with the patient and blurs the boundaries of the relationship. A caring nurse will acknowledge the patient's gesture of appreciation, but the gift should not be accepted.

6. A patient discloses several concerns and associated feelings. If the nurse wants to seek clarification, which comment would be appropriate? a. "What are the common elements here?" b. "Tell me again about your experiences." c. "Am I correct in understanding that...?" d. "Tell me everything from the beginning."

ANS: C Asking, "Am I correct in understanding that...?" permits clarification to ensure that both the nurse and patient share mutual understanding of the communication. Asking about common elements encourages comparison rather than clarification. The remaining responses are implied questions that suggest the nurse was not listening.

7. A patient tells the nurse, "I don't think I'll ever get out of here." Select the nurse's most therapeutic response. a. "Don't talk that way. Of course you will leave here!" b. "Keep up the good work and you certainly will." c. "You don't think you're making progress?" d. "Everyone feels that way sometimes."

ANS: C By asking if the patient does not believe that progress has been made, the nurse is reflecting by putting into words what the patient is hinting. By making communication more explicit, issues are easier to identify and resolve. The remaining options are nontherapeutic techniques. Telling the patient not to "talk that way" is disapproving. Saying that everyone feels that way at times minimizes feelings. Telling the patient that good work will always result in success is falsely reassuring.

During which phase of the nurse-patient relationship can the nurse anticipate that identified patient issues will be explored and resolved? a. Preorientation b. Orientation c. Working d. Termination

ANS: C During the working phase, the nurse strives to assist the patient in making connections among dysfunctional behaviors, thinking, and emotions and offers support while alternative coping behaviors are tried.

A patient says, "I'm still on restriction, but I want to attend some off-unit activities. Would you ask the doctor to change my privileges?" What is the nurse's best response? a. "Why are you asking me when you're able to speak for yourself?" b. "I will be glad to address it when I see your doctor later today." c. "That's a good topic for you to discuss with your doctor." d. "Do you think you can't speak to a doctor?"

ANS: C Nurses should encourage patients to work at their optimal level of functioning. A nurse does not act for the patient unless it is necessary. Acting for a patient increases feelings of helplessness and dependency.

Which issues should a nurse address during the first interview with a patient with a psychiatric disorder? a. Trust, congruence, attitudes, and boundaries b. Goals, resistance, unconscious motivations, and diversion c. Relationship parameters, the contract, confidentiality, and termination d. Transference, countertransference, intimacy, and developing resources

ANS: C Relationship parameters, the contract, confidentiality, and termination are issues that should be considered during the orientation phase of the relationship. The remaining options are issues that are dealt with later.

17. Which principle should guide the nurse in determining the extent of silence to use during patient interview sessions? a. Nurses are responsible for breaking silences. b. Patients withdraw if silences are prolonged. c. Silence provides meaningful moments for reflection. d. Silence helps patients know that what they have said is understood.

ANS: C Silence can be helpful to both participants by giving each an opportunity to contemplate what has transpired, weigh alternatives, and formulate ideas. A nurse breaking silences is not a principle related to silences. Saying that patients withdraw during long silences or that silence helps patients know that they are understood are both inaccurate statements. Feedback helps patients know they have been understood.

*After several therapeutic encounters with a patient who recently attempted suicide, which occurrence should cause the nurse to consider the possibility of countertransference?* a. The patient's reactions toward the nurse seem realistic and appropriate. b. The patient states, "Talking to you feels like talking to my parents." c. The nurse feels unusually happy when the patient's mood begins to lift. d. The nurse develops a trusting relationship with the patient.

ANS: C Strong positive or negative reactions toward a patient or over-identification with the patient indicate possible countertransference. Nurses must carefully monitor their own feelings and reactions to detect countertransference and then seek supervision. Realistic and appropriate reactions from a patient toward a nurse are desirable. One incorrect response suggests transference. A trusting relationship with the patient is desirable.

*Termination of a therapeutic nurse-patient relationship has been successful when the nurse:* a. avoids upsetting the patient by shifting focus to other patients before the discharge. b. gives the patient a personal telephone number and permission to call after discharge. c. discusses with the patient changes that happened during the relationship and evaluates outcomes. d. offers to meet the patient for coffee and conversation three times a week after discharge.

ANS: C Summarizing and evaluating progress help validate the experience for the patient and the nurse and facilitate closure. Termination must be discussed; avoiding discussion by spending little time with the patient promotes feelings of abandonment. Successful termination requires that the relationship be brought to closure without the possibility of dependency-producing ongoing contact.

8. Documentation in a patient's chart shows, "Throughout a 5-minute interaction, patient fidgeted and tapped left foot, periodically covered face with hands, and looked under chair while stating, 'I enjoy spending time with you.'" Which analysis is most accurate? a. Patient is giving positive feedback about the nurse's communication techniques. b. Nurse is viewing the patient's behavior through a cultural filter. c. Patient's verbal and nonverbal messages are incongruent. d. Patient is demonstrating psychotic behaviors.

ANS: C When a verbal message is not reinforced with nonverbal behavior, the message is confusing and incongruent. Some clinicians call it a "mixed message." It is inaccurate to say that the patient is giving positive feedback about the nurse's communication techniques. The concept of a cultural filter is not relevant to the situation; a cultural filter determines what a person will pay attention to and what he or she will ignore. Data are insufficient to draw the conclusion that the patient is demonstrating psychotic behaviors.

The desired outcome for a patient experiencing insomnia is, "Patient will sleep for a minimum of 5 hours nightly within 7 days." At the end of 7 days, review of sleep data shows the patient sleeps an average of 4 hours nightly and takes a 2-hour afternoon nap. The nurse will document the outcome as: a. consistently demonstrated. b. often demonstrated. c. sometimes demonstrated. d. never demonstrated.

ANS: D Although the patient is sleeping 6 hours daily, the total is not one uninterrupted session at night. Therefore, the outcome must be evaluated as never demonstrated. See relationship to audience response question.

Nursing behaviors associated with the implementation phase of nursing process are concerned with: a. participating in mutual identification of patient outcomes. b. gathering accurate and sufficient patient-centered data. c. comparing patient responses and expected outcomes. d. carrying out interventions and coordinating care.

ANS: D Nursing behaviors relating to implementation include using available resources, performing interventions, finding alternatives when necessary, and coordinating care with other team members.

The desired outcome for a patient experiencing insomnia is, "Patient will sleep for a minimum of 5 hours nightly within 7 days." At the end of 7 days, review of sleep data shows the patient sleeps an average of 4 hours nightly and takes a 2-hour afternoon nap. What is the nurse's next action? a. Continue the current plan without changes. b. Remove this nursing diagnosis from the plan of care. c. Write a new nursing diagnosis that better reflects the problem. d. Examine interventions for possible revision of the target date.

ANS: D Sleeping a total of 5 hours at night remains a reasonable outcome. Extending the period for attaining the outcome may be appropriate. Examining interventions might result in planning an activity during the afternoon rather than permitting a nap. Continuing the current plan without changes is inappropriate. Removing this nursing diagnosis from the plan of care would be correct when the outcome was met and the problem resolved. Writing a new nursing diagnosis is inappropriate because no other nursing diagnosis relates to the problem.

A nurse documents: "Patient is mute despite repeated efforts to elicit speech. Makes no eye contact. Inattentive to staff. Gazes off to the side or looks upward rather than at speaker." Which nursing diagnosis should be considered? a. Defensive coping b. Decisional conflict c. Risk for other-directed violence d. Impaired verbal communication

ANS: D The defining characteristics are more related to the nursing diagnosis of impaired verbal communication than to the other nursing diagnoses.

A nurse prepares to assess a new patient who moved to the United States from Central America three years ago. After introductions, what is the nurse's next comment? a. "How did you get to the United States?" b. "Would you like for a family member to help you talk with me?" c. "An interpreter is available. Would you like for me to make a request for these services?" d. "Are you comfortable conversing in English, or would you prefer to have a translator present?"

ANS: D The nurse should determine whether a translator is needed by first assessing the patient for language barriers. Accuracy of the assessment depends on the ability to communicate in a language that is familiar to the patient. Family members are not always reliable translators. An interpreter may change the patient's responses; a translator is a better resource.

A patient states, "I'm not worth anything. I have negative thoughts about myself. I feel anxious and shaky all the time. Sometimes I feel so sad that I want to go to sleep and never wake up." Which nursing intervention should have the highest priority? a. Self-esteem-building activities b. Anxiety self-control measures c. Sleep enhancement activitie d. Suicide precautions

ANS: D The nurse would place a priority on monitoring and reinforcing suicide self-restraint because it relates directly and immediately to patient safety. Patient safety is always a priority concern. The nurse should monitor and reinforce all patient attempts to control anxiety, improve sleep patterns, and develop self-esteem, while giving priority attention to suicide self-restraint.

Select the best outcome for a patient with the nursing diagnosis: Impaired social interaction related to sociocultural dissonance as evidenced by stating, "Although I'd like to, I don't join in because I don't speak the language very well." Patient will: a. show improved use of language. b. demonstrate improved social skills. c. become more independent in decision making. d. select and participate in one group activity per day.

ANS: D The outcome describes social involvement on the part of the patient. Neither cooperation nor independence has been an issue. The patient has already expressed a desire to interact with others. Outcomes must be measurable. Two of the distracters are not measurable.

Which statement made by a patient during an initial assessment interview should serve as the priority focus for the plan of care? a. "I can always trust my family." b. "It seems like I always have bad luck." c. "You never know who will turn against you." d. "I hear evil voices that tell me to do bad things."

ANS: D The statement regarding evil voices tells the nurse that the patient is experiencing auditory hallucinations and may create risks for violence. The other statements are vague and do not clearly identify the patient's chief symptom.

At what point in an assessment interview would a nurse ask, "How does your faith help you in stressful situations?" During the assessment of: a. childhood growth and development b. substance use and abuse c. educational background d. coping strategies

ANS: D When discussing coping strategies, the nurse might ask what the patient does when upset, what usually relieves stress, and to whom the patient goes to talk about problems. The question regarding whether the patient's faith helps deal with stress fits well here. It would be out of place if introduced during exploration of the other topics.

A nurse assesses an older adult patient brought to the emergency department by a family member. The patient was wandering outside saying, "I can't find my way home." The patient is confused and unable to answer questions. Select the nurse's best action. a. Record the patient's answers to questions on the nursing assessment form. b. Ask an advanced practice nurse to perform the assessment interview. c. Call for a mental health advocate to maintain the patient's rights. d. Obtain important information from the family member.

ANS: D When the patient (primary source) is unable to provide information, secondary sources should be used, in this case, the family member. Later, more data may be obtained from other information sources familiar with the patient. An advanced practice nurse is not needed for this assessment; it is within the scope of practice of the staff nurse. Calling a mental health advocate is unnecessary. See relationship to audience response question.

4. A nurse interacts with a newly hospitalized patient. Select the nurse's comment that applies the communication technique of "offering self." a. "I've also had traumatic life experiences. Maybe it would help if I told you about them." b. "Why do you think you had so much difficulty adjusting to this change in your life?" c. "I hope you will feel better after getting accustomed to how this unit operates." d. "I'd like to sit with you for a while to help you get comfortable talking to me."

ANS: D "Offering self" is a technique that should be used in the orientation phase of the nurse-patient relationship. Sitting with the patient, an example of "offering self," helps build trust and conveys that the nurse cares about the patient. Two incorrect responses are ineffective and nontherapeutic. The other incorrect response is therapeutic but an example of "offering hope."

15. A Puerto Rican-American patient uses dramatic body language when describing emotional discomfort. Which analysis most likely explains the patient's behavior? The patient: a. Has a histrionic personality disorder. b. Believes dramatic body language is sexually appealing. c. Wishes to impress staff with the degree of emotional pain. d. Belongs to a culture in which dramatic body language is the norm.

ANS: D Members of Hispanic-American subcultures tend to use high affect and dramatic body language as they communicate. The other options are more remote possibilities.

13. A Filipino-American patient had this nursing diagnosis: Situational low self-esteem, related to poor social skills as evidenced by lack of eye contact. Interventions were used to raise the patient's self-esteem; however, after 3 weeks, the patient's eye contact did not improve. What is the most accurate analysis of this scenario? a. The patient's eye contact should have been directly addressed by role-playing to increase comfort with eye contact. b. The nurse should not have independently embarked on assessment, diagnosis, and planning for this patient. c. The patient's poor eye contact is indicative of anger and hostility that remain unaddressed. d. The nurse should have assessed the patient's culture before making this diagnosis and plan.

ANS: D The amount of eye contact in which a person engages is often culturally determined. In some cultures, eye contact is considered insolent, whereas in other cultures, eye contact is expected. Asian Americans, including persons from the Philippines, often prefer not to engage in direct eye contact.

A nurse caring for a withdrawn, suspicious patient recognizes development of feelings of anger toward the patient. The nurse should: a. suppress the angry feelings. b. express the anger openly and directly with the patient. c. tell the nurse manager to assign the patient to another nurse. d. discuss the anger with a clinician during a supervisory session.

ANS: D The nurse is accountable for the relationship. Objectivity is threatened by strong positive or negative feelings toward a patient. Supervision is necessary to work through countertransference feelings.

1. A patient says to the nurse, "I dreamed I was stoned. When I woke up, I felt emotionally drained, as though I hadn't rested well." Which comment would be appropriate if the nurse seeks clarification? a. "It sounds as though you were uncomfortable with the content of your dream." b. "I understand what you're saying. Bad dreams leave me feeling tired, too." c. "So, all in all, you feel as though you had a rather poor night's sleep?" d. "Can you give me an example of what you mean by 'stoned'?"

ANS: D The technique of clarification is therapeutic and helps the nurse examine the meaning of the patient's statement. Asking for a definition of "stoned" directly asks for clarification. Restating that the patient is uncomfortable with the dream's content is parroting, a nontherapeutic technique. The other responses fail to clarify the meaning of the patient's comment.

Which remark by a patient indicates passage from orientation to the working phase of a nurse-patient relationship? a. "I don't have any problems." b. "It is so difficult for me to talk about problems." c. "I don't know how it will help to talk to you about my problems." d. "I want to find a way to deal with my anger without becoming violent."

ANS: D Thinking about a more constructive approach to dealing with anger indicates a readiness to make a behavioral change. Behavioral change is associated with the working phase of the relationship. Denial is often seen in the orientation phase. It is common early in the relationship, before rapport and trust are firmly established, for a patient to express difficulty in talking about problems. Stating skepticism about the effectiveness of the nurse-patient relationship is more typically a reaction during the orientation phase.

A patient says, "People should be allowed to commit suicide without interference from others." A nurse replies, "You're wrong. Nothing is bad enough to justify death." What is the best analysis of this interchange? a. The patient is correct. b. The nurse is correct. c. Neither person is correct. d. Differing values are reflected in the two statements.

ANS: D Values guide beliefs and actions. The individuals stating their positions place different values on life and autonomy. Nurses must be aware of their own values and be sensitive to the values of others.

16. During an interview, a patient attempts to shift the focus from self to the nurse by asking personal questions. The nurse should respond by saying: a. "You've turned the tables on me." b. "Nurses direct the interviews with patients." c. "Do not ask questions about my personal life." d. "The time we spend together is to discuss your concerns."

ANS: D When a patient tries to focus on the nurse, the nurse should refocus the discussion back onto the patient. Telling the patient that interview time should be used to discuss patient concerns refocuses discussion in a neutral way. Telling patients not to ask about the nurse's personal life shows indignation. Saying that nurses prefer to direct the interview reflects superiority. Saying "You've turned the tables on me" states the fact but does not refocus the interview.

A nurse is talking with a patient, and 5 minutes remain in the session. The patient has been silent most of the session. Another patient comes to the door of the room, interrupts, and says to the nurse, "I really need to talk to you." The nurse should: a. invite the interrupting patient to join in the session with the current patient. b. say to the interrupting patient, "I am not available to talk with you at the present time." c. end the unproductive session with the current patient and spend time with the interrupting patient. d. tell the interrupting patient, "This session is 5 more minutes; then I will talk with you."

ANS: D When a specific duration for sessions has been set, the nurse must adhere to the schedule. Leaving the first patient would be equivalent to abandonment and would destroy any trust the patient had in the nurse. Adhering to the contract demonstrates that the nurse can be trusted and that the patient and the sessions are important. The incorrect responses preserve the nurse-patient relationship with the silent patient but may seem abrupt to the interrupting patient, abandon the silent patient, or fail to observe the contract with the silent patient.

The nurse records this entry in a patient's progress notes: Patient escorted to unit by ER nurse at 2130. Patient's clothing was dirty. In interview room, patient sat with hands over face, sobbing softly. Did not acknowledge nurse or reply to questions. After several minutes, abruptly arose, ran to window, and pounded. Shouted repeatedly, "Let me out of here." Verbal intervention unsuccessful. Order for stat dose 2 mg haloperidol PO obtained; medication administered at 2150. By 2215, patient stopped shouting and returned to sit wordlessly in chair. Patient placed on one-to-one observation. How should this documentation be evaluated? a. Uses unapproved abbreviations b. Contains subjective material c. Too brief to be of value d. Excessively wordy e. Meets standards

ANS: E This narrative note describes patient appearance, behavior, and conversation. It mentions that less-restrictive measures were attempted before administering medication and documents patient response to medication. This note would probably meet standards. A complete nursing assessment would be in order as soon as the patient is able to participate. Subjective material is absent from the note. Abbreviations are acceptable.

A psychiatric nurse best applies the ethical principle of autonomy by: a. exploring alternative solutions with a patient, who then makes a choice. b. suggesting that two patients who were fighting be restricted to the unit. c. intervening when a self-mutilating patient attempts to harm self. d. staying with a patient demonstrating a high level of anxiety.

ANS:A Autonomy is the right to self-determination, that is, to make one's own decisions. By exploring alternatives with the patient, the patient is better equipped to make an informed, autonomous decision. The distracters demonstrate beneficence, fidelity, and justice. PTS:1 DIF: Cognitive Level: Apply (Application) REF: Page 99 TOP: Nursing Process: Evaluation MSC: Client Needs: Safe, Effective Care Environment

A patient in alcohol rehabilitation reveals to the nurse, "I feel terrible guilt for sexually abusing my 6-year-old before I was admitted. " Select the nurse's most important action. a. Anonymously report the abuse by phone to the local child protection agency. b. Reply, "I'm glad you feel comfortable talking to me about it." c. File a written report with the agency's ethics committee. d. Respect nurse-patient relationship confidentiality.

ANS:A Laws regarding child abuse reporting discovered by a professional during the suspected abuser's alcohol or drug treatment differ by state. Federal law supersedes state law and prohibits disclosure without a court order except in instances in which the report can be made anonymously or without identifying the abuser as a patient in an alcohol or drug treatment facility. PTS:1 DIF: Cognitive Level: Apply (Application) REF: Page 106-107 TOP: Nursing Process: Implementation MSC: Client Needs: Safe, Effective Care Environment

A nurse is concerned that an agency's policies are inadequate. Which understanding about the relationship between substandard institutional policies and individual nursing practice should guide nursing practice? a. Agency policies do not exempt an individual nurse of responsibility to practice according to professional standards of nursing care. b. Agency policies are the legal standard by which a professional nurse must act and therefore override other standards of care. c. Faced with substandard policies, a nurse has a responsibility to inform the supervisor and discontinue patient care immediately. d. Interpretation of policies by the judicial system is rendered on an individual basis and therefore cannot be predicted.

ANS:A Nurses are professionally bound to uphold standards of practice regardless of lesser standards established by a health care agency or a state. Conversely, if the agency standards are higher than standards of practice, the agency standards must be upheld. The courts may seek to establish the standard of care through the use of expert witnesses when the issue is clouded. PTS:1 DIF: Cognitive Level: Understand (Comprehension) REF: Page 108-109 TOP: Nursing Process: Planning MSC: Client Needs: Safe, Effective Care Environment

In order to release information to another health care facility or third party regarding a patient diagnosed with a mental illness, the nurse must obtain: a. a signed consent by the patient for release of information stating specific information to be released. b. a verbal consent for information release from the patient and the patient's guardian or next of kin. c. permission from members of the health care team who participate in treatment planning. d. approval from the attending psychiatrist to authorize the release of information.

ANS:A Nurses have an obligation to protect patients' privacy and confidentiality. Clinical information should not be released without the patient's signed consent for the release. PTS:1 DIF: Cognitive Level: Apply (Application) REF: Page 104-106 TOP: Nursing Process: Planning MSC: Client Needs: Safe, Effective Care Environment

A person in the community asks, "Why aren't people with mental illness kept in state institutions anymore?" Select the nurse's best response. a. "Less restrictive settings are available now to care for individuals with mental illness." b. "There are fewer persons with mental illness, so less hospital beds are needed. " c. "Most people with mental illness are still in psychiatric institutions." d. "Psychiatric institutions violated patients' rights."

ANS:A The community is a less restrictive alternative than hospitals for treatment of persons with mental illness. The distracters are incorrect and part of the stigma of mental illness. PTS:1 DIF: Cognitive Level: Apply (Application) REF: Page 100 TOP: Nursing Process: Implementation MSC: Client Needs: Safe, Effective Care Environment

Insurance will not pay for continued private hospitalization of a mentally ill patient. The family considers transferring the patient to a public hospital but expresses concern that the patient will not get any treatment if transferred. Select the nurse's most helpful reply. a. "By law, treatment must be provided. Hospitalization without treatment violates patients' rights." b. "All patients in public hospitals have the right to choose both a primary therapist and a primary nurse." c. "You have a justifiable concern because the right to treatment extends only to provision of food, shelter, and safety." d. "Much will depend on other patients, because the right to treatment for a psychotic patient takes precedence over the right to treatment of a patient who is stable."

ANS:A The right to medical and psychiatric treatment was conferred on all patients hospitalized in public mental hospitals with the enactment of the federal Hospitalization of Mentally Ill Act in1964. PTS:1 DIF: Cognitive Level: Understand (Comprehension) REF: Page 101-102 TOP: Nursing Process: Implementation MSC: Client Needs: Safe, Effective Care Environment

Select the example of a tort. a. The plan of care for a patient is not completed within 24 hours of the patient's admission. b. A nurse gives a PRN dose of an antipsychotic drug to an agitated patient because the unit is short-staffed. c. An advanced practice nurse recommends hospitalization for a patient who is dangerous to self and others. d. A patient's admission status changed from involuntary to voluntary after the patient's hallucinations subside.

ANS:B A tort is a civil wrong against a person that violates his or her rights. Giving unnecessary medication for the convenience of staff controls behavior in a manner similar to secluding a patient; thus, false imprisonment is a possible charge. The other options do not exemplify torts. PTS:1 DIF: Cognitive Level: Understand (Comprehension) REF: Page 108-109 TOP: Nursing Process: Evaluation MSC: Client Needs: Safe, Effective Care Environment

An adolescent hospitalized after a violent physical outburst tells the nurse, "I'm going to kill my father, but you can't tell anyone." Select the nurse's best response. a. "You are right. Federal law requires me to keep clinical information private." b. "I am obligated to share that information with the treatment team." c. "Those kinds of thoughts will make your hospitalization longer." d. "You should share this thought with your psychiatrist."

ANS:B Breach of nurse-patient confidentiality does not pose a legal dilemma for nurses in these circumstances because a team approach to delivery of psychiatric care presumes communication of patient information to other staff members to develop treatment plans and outcome criteria. The patient should also know that the team has a duty to warn the father of the risk for harm. PTS:1 DIF: Cognitive Level: Apply (Application) REF: Page 104-106 TOP: Nursing Process: Implementation MSC: Client Needs: Safe, Effective Care Environment

Which nursing intervention demonstrates false imprisonment? a. A confused and combative patient says, "I'm getting out of here, and no one can stop me." The nurse restrains this patient without a health care provider's order and then promptly obtains an order. b. A patient has been irritating and attention-seeking much of the day. A nurse escorts the patient down the hall saying, "Stay in your room, or you'll be put in seclusion." c. An involuntarily hospitalized patient with suicidal ideation runs out of the psychiatric unit. The nurse rushes after the patient and convinces the patient to return to the unit. d. An involuntarily hospitalized patient with homicidal ideation attempts to leave the facility. A nurse calls the security team and uses established protocols to prevent the patient from leaving.

ANS:B False imprisonment involves holding a competent person against his or her will. Actual force is not a requirement of false imprisonment. The individual needs only to be placed in fear of imprisonment by someone who has the ability to carry out the threat. If a patient is not competent (confused), then the nurse should act with beneficence. Patients admitted involuntarily should not be allowed to leave without permission of the treatment team. PTS:1 DIF: Cognitive Level: Apply (Application) REF: Page 106 (Box 6-3) | Page 108 | Page 109 (Box 6-4) TOP: Nursing Process: Evaluation MSC: Client Needs: Safe, Effective Care Environment

A patient diagnosed with schizophrenia believes a local minister stirred evil spirits. The patient threatens to bomb a local church. The psychiatrist notifies the minister. Select the answer with the correct rationale. The psychiatrist: a. released information without proper authorization. b. demonstrated the duty to warn and protect. c. violated the patient's confidentiality. d. avoided charges of malpractice.

ANS:B It is the health care professional's duty to warn or notify an intended victim after a threat of harm has been made. Informing a potential victim of a threat is a legal responsibility of the health care professional. It is not a violation of confidentiality. PTS:1 DIF: Cognitive Level: Understand (Comprehension) REF: Page 108-110 TOP: Nursing Process: Implementation MSC: Client Needs: Safe, Effective Care Environment

Two hospitalized patients fight whenever they are together. During a team meeting, a nurse asserts that safety is of paramount importance, so treatment plans should call for both patients to be secluded to keep them from injuring each other. This assertion: a. reinforces the autonomy of the two patients. b. violates the civil rights of both patients. c. represents the intentional tort of battery. d. correctly places emphasis on safety.

ANS:B Patients have a right to treatment in the least restrictive setting. Safety is important, but less restrictive measures should be tried first. Unnecessary seclusion may result in a charge of false imprisonment. Seclusion violates the patient's autonomy. The principle by which the nurse is motivated is beneficence, not justice. The tort represented is false imprisonment. PTS:1 DIF: Cognitive Level: Apply (Application) REF: Page 100 | Page 103-104 TOP: Nursing Process: Planning MSC: Client Needs: Safe, Effective Care Environment

A patient experiencing psychosis asks a psychiatric technician, "What's the matter with me?" The technician replies, "Nothing is wrong with you. You just need to use some self-control." The nurse who overheard the exchange should take action based on: a. the technician's unauthorized disclosure of confidential clinical information. b. violation of the patient's right to be treated with dignity and respect. c. the nurse's obligation to report caregiver negligence. d. the patient's right to social interaction.

ANS:B Patients have the right to be treated with dignity and respect. The technician's comment disregards the seriousness of the patient's illness. The Code of Ethics for Nurses requires intervention. Patient emotional abuse has been demonstrated, not negligence. An interaction with the technician is not an aspect of social interaction. The technician did not disclose clinical information. PTS:1 DIF: Cognitive Level: Understand (Comprehension) REF: Page 105 (Box 6-2) TOP: Nursing Process: Evaluation MSC: Client Needs: Psychosocial Integrity

A nurse prepares to administer a scheduled injection of haloperidol decanoate (Haldol depot) to an outpatient with schizophrenia. As the nurse swabs the site, the patient shouts, "Stop! I don't want to take that medicine anymore. I hate the side effects." Select the nurse's best action. a. Assemble other staff for a show of force and proceed with the injection, using restraint if necessary. b. Stop the medication administration procedure and say to the patient, "Tell me more about the side effects you've been having." c. Proceed with the injection but explain to the patient that there are medications thatwill help reduce the unpleasant side effects. d. Say to the patient, "Since I've already drawn the medication in the syringe, I'm required to give it, but let's talk to the doctor about delaying next month's dose."

ANS:B Patients with mental illness retain their civil rights unless there is clear, cogent, and convincing evidence of dangerousness. The patient in this situation presents no evidence of dangerousness. The nurse, as an advocate and educator, should seek more information about the patient's decision and not force the medication. PTS:1 DIF: Cognitive Level: Apply (Application) REF: Page 99-100 | Page 102-103 TOP: Nursing Process: Implementation MSC: Client Needs: Safe, Effective Care Environment

Which action by a nurse constitutes a breach of a patient's right to privacy? a. Documenting the patient's daily behavior during hospitalization b. Releasing information to the patient's employer without consent c. Discussing the patient's history with other staff during care planning d. Asking family to share information about a patient's pre-hospitalization behavior

ANS:B Release of information without patient authorization violates the patient's right to privacy. The other options are acceptable nursing practices. See relationship to audience response question. PTS:1 DIF: Cognitive Level: Apply (Application) REF: Page 104-106 TOP: Nursing Process: Evaluation MSC: Client Needs: Safe, Effective Care Environment

A nurse finds a psychiatric advance directive in the medical record of a patient experiencing psychosis. The directive was executed during a period when the patient was stable and competent. The nurse should: a. review the directive with the patient to ensure it is current. b. ensure that the directive is respected in treatment planning. c. consider the directive only if there is a cardiac or respiratory arrest. d. encourage the patient to revise the directive in light of the current health problem.

ANS:B The nurse has an obligation to honor the right to self-determination. An advanced psychiatric directive supports that goal. Since the patient is currently psychotic, the terms of the directive now apply. PTS:1 DIF: Cognitive Level: Apply (Application) REF: Page 103-104 TOP: Nursing Process: Implementation MSC: Client Needs: Safe, Effective Care Environment

An aide in a psychiatric hospital says to the nurse, "We don't have time every day to help each patient complete a menu selection. Let's tell dietary to prepare popular choices and send them to our unit." Select the nurse's best response. a. "Thanks for the suggestion, but that idea may not work because so many patients take MAOI (monoamine oxidase inhibitor) antidepressants." b. "Thanks for the idea, but it's important to treat patients as individuals. Giving choices is one way we can respect patients' individuality." c. "Thank you for the suggestion, but the patients' bill of rights requires us to allow patients to select their own diet." d. "Thank you. That is a very good idea. It will make meal preparation easier for the dietary department."

ANS:B The nurse's response to the aide should recognize patients' rights to be treated with dignity and respect as well as promote autonomy. This response also shows respect for the aide and fulfills the nurse's obligation to provide supervision of unlicensed personnel. The incorrect responses have flawed rationale or do not respect patients as individuals. PTS:1 DIF: Cognitive Level: Analyze (Analysis) REF: Page 99 | Page 105 (Box 6-2) TOP: Nursing Process: Implementation MSC: Client Needs: Safe, Effective Care Environment

A voluntarily hospitalized patient tells the nurse, "Get me the forms for discharge. I want to leave now." Select the nurse's best response. a. "I will get the forms for you right now and bring them to your room." b. "Since you signed your consent for treatment, you may leave if you desire." c. "I will get them for you, but let's talk about your decision to leave treatment." d. "I cannot give you those forms without your health care provider's permission."

ANS:C A voluntarily admitted patient has the right to demand and obtain release in most states. However, as a patient advocate, the nurse is responsible for weighing factors related to the patient's wishes and best interests. By asking for information, the nurse may be able to help the patient reconsider the decision. Facilitating discharge without consent is not in the patient's best interests before exploring the reason for the request. PTS:1 DIF: Cognitive Level: Apply (Application) REF: Page 100-101 | Page 109 (Box 6-4) TOP: Nursing Process: Implementation MSC: Client Needs: Safe, Effective Care Environment

After leaving work, a nurse realizes documentation of administration of a PRN medication was omitted. This off-duty nurse phones the nurse on duty and says, "Please document administration of the medication for me. My password is alpha1." The nurse receiving the call should: a. fulfill the request promptly. b. document the caller's password. c. refer the matter to the charge nurse to resolve. d. report the request to the patient's health care provider.

ANS:C Fraudulent documentation may be grounds for discipline by the state board of nursing. Referring the matter to the charge nurse will allow observance of hospital policy while ensring that documentation occurs. Notifying the health care provider would be unnecessary when the charge nurse can resolve the problem. Nurses should not provide passwords to others. PTS:1 DIF: Cognitive Level: Apply (Application) REF: Page 110-112 TOP: Nursing Process: Implementation MSC: Client Needs: Safe, Effective Care Environment

Which individual diagnosed with a mental illness may need involuntary hospitalization? An individual: a. who has a panic attack after her child gets lost in a shopping mall b. with visions of demons emerging from cemetery plots throughout the community c. who takes 38 acetaminophen tablets after the person's stock portfolio becomes worthless d. diagnosed with major depression who stops taking prescribed antidepressant medication

ANS:C Involuntary hospitalization protects patients who are dangerous to themselves or others and cannot care for their own basic needs. Involuntary hospitalization also protects other individuals in society. An overdose of acetaminophen indicates dangerousness to self. The behaviors described in the other options are not sufficient to require involuntary hospitalization. PTS:1 DIF: Cognitive Level: Apply (Application) REF: Page 100-101 TOP: Nursing Process: Assessment MSC: Client Needs: Safe, Effective Care Environment

A new antidepressant is prescribed for an elderly patient with major depression, but the dose is more than the usual geriatric dose. The nurse should: a. consult a reliable drug reference. b. teach the patient about possible side effects and adverse effects. c. withhold the medication and confer with the health care provider. d. encourage the patient to increase oral fluids to reduce drug concentration.

ANS:C The dose of antidepressants for elderly patients is often less than the usual adult dose. The nurse should withhold the medication and consult the health care provider who wrote the order. The nurse's duty is to practice according to professional standards as well as intervene and protect the patient. PTS:1 DIF: Cognitive Level: Apply (Application) REF: Page 108-110 TOP: Nursing Process: Implementation MSC: Clint Needs: Safe, Effective Care Environment

In which situations would a nurse have the duty to intervene and report? Select all that apply. a. A peer has difficulty writing measurable outcomes. b. A health care provider gives a telephone order for medication. c. A peer tries to provide patient care in an alcohol-impaired state. d. A team member violates relationship boundaries with a patient. e. A patient refuses medication prescribed by a licensed health care provider.

ANS:C, D Both keyed answers are events that jeopardize patient safety. The distracters describe situations that may be resolved with education or that are acceptable practices. PTS:1 DIF: Cognitive Level: Apply (Application) REF: Page 105 (Box 6-2) | Page 108-109 TOP: Nursing Process: Evaluation MSC: Client Needs: Safe, Effective Care Environment

Which actions violate the civil rights of a psychiatric patient? The nurse: (select all that apply) a. performs mouth checks after overhearing a patient say, "I've been spitting out my medication." b. begins suicide precautions before a patient is assessed by the health care provider. c. opens and reads a letter a patient left at the nurse's station to be mailed. d. places a patient's expensive watch in the hospital business office safe. e. restrains a patient who uses profanity when speaking to the nurse.

ANS:C, E

A newly admitted acutely psychotic patient is a private patient of the medical director and a private-pay patient. To whom does the psychiatric nurse assigned to the patient owe the duty of care? a. Medical director c. Profession b. Hospital d. Patient

ANS:D Although the nurse is accountable to the health care provider, the agency, the patient, and the profession, the duty of care is owed to the patient. PTS:1 DIF: Cognitive Level: Understand (Comprehension) REF: Page 108 TOP: Nursing Process: Implementation MSC: Client Needs: Safe, Effective Care Environment

What is the legal significance of a nurse's action when a patient verbally refuses medication and the nurse gives the medication over the patient's objection? The nurse: a. has been negligent. c. fulfilled the standard of care. b. committed malpractice. d. can be charged with battery.

ANS:D Battery is an intentional tort in which one individual violates the rights of another through touching without consent. Forcing a patient to take medication after the medication was refused constitutes battery. The charge of battery can be brought against the nurse. The medication may not necessarily harm the patient; harm is a component of malpractice. PTS:1 DIF: Cognitive Level: Understand (Comprehension) REF: Page 108-109 TOP: Nursing Process: Evaluation MSC: Client Needs: Safe, Effective Care Environment

A patient with psychosis became aggressive, struck another patient, and required seclusion. Select the best documentation. a. Patient struck another patient who attempted to leave day room to go to bathroom. Seclusion necessary at 1415. Plan: Maintain seclusion for 8 hours and keep these two patients away from each other for 24 hours. b. Seclusion ordered by physician at 1415 after command hallucinations told the patient to hit another patient. Careful monitoring of patient maintained during period of seclusion. c. Seclusion ordered by MD for aggressive behavior. Begun at 1415. Maintained for 2 hours without incident. Outcome: Patient calmer and apologized for outburst. d. Patient pacing, shouting. Haloperidol 5 mg given PO at 1300. No effect by 1315. At 1415 patient yelled, "I'll punch anyone who gets near me," and struck another patient with fist. Physically placed in seclusion at 1420. Seclusion order obtained from MD at 1430.

ANS:D Documentation must be specific and detail the key aspects of care. It should demonstrate implementation of the least restrictive alternative. Justification for why a patient was secluded should be recorded, along with interventions attempted in an effort to avoid seclusion. Documentation should include a description of behavior and verbalizations, interventions tried and their outcomes, and the name of the health care provider ordering the use of seclusion. PTS:1 DIF: Cognitive Level: Apply (Application) REF: Page 103-104 | Page 110-112 TOP: Nursing Process: Implementation MSC: Client Needs: Safe, Effective Care Environment

Which patient meets criteria for involuntary hospitalization for psychiatric treatment? The patient who: a. is noncompliant with the treatment regimen. b. fraudulently files for bankruptcy. c. sold and distributed illegal drugs. d. threatens to harm self and others.

ANS:D Involuntary hospitalization protects patients who are dangerous to themselves or others and cannot care for their own basic needs. Involuntary commitment also protects other individuals in society. The behaviors described in the other options are not sufficient to require involuntary hospitalization. PTS:1 DIF: Cognitive Level: Understand (Comprehension) REF: Page 100-101 TOP: Nursing Process: Assessment MSC: Client Needs: Safe, Effective Care Environment

Which documentation of a patient's behavior best demonstrates a nurse's observations? a. Isolates self from others. Frequently fell asleep during group. Vital signs stable. b. Calmer; more cooperative. Participated actively in group. No evidence of psychotic thinking. c. Appeared to hallucinate. Frequently increased volume on television, causing conflict with others. d. Wore four layers of clothing. States, "I need protection from evil bacteria trying to pierce my skin."

ANS:D The documentation states specific observations of the patient's appearance and the exact statements made. The other options are vague or subjective statements and can be interpreted in different ways. PTS:1 DIF: Cognitive Level: Apply (Application) REF: Page 110-112 TOP: Nursing Process: Implementation MSC: Client Needs: Safe, Effective Care Environment

A family member of a patient with delusions of persecution asks the nurse, "Are there any circumstances under which the treatment team is justified in violating a patient's right to confidentiality?" The nurse should reply that confidentiality may be breached: a. under no circumstances. b. at the discretion of the psychiatrist. c. when questions are asked by law enforcement. d. if the patient threatens the life of another person.

ANS:D The duty to warn a person whose life has been threatened by a psychiatric patient overrides the patient's right to confidentiality. The right to confidentiality is not suspended at the discretion of the therapist or for legal investigations. PTS:1 DIF: Cognitive Level: Understand (Comprehension) REF: Page 106 TOP: Nursing Process: Implementation MSC: Client Needs: Safe, Effective Care Environment

In a team meeting a nurse says, "I'm concerned about whether we are behaving ethically by using restraint to prevent one patient from self-mutilation, while the care plan for another self-mutilating patient requires one-on-one supervision." Which ethical principle most clearly applies to this situation? a. Beneficence c. Fidelity b. Autonomy d. Justice

ANS:D The nurse is concerned about justice, that is, fair distribution of care, which includes treatment with the least restrictive methods for both patients. Beneficence means promoting the good of others. Autonomy is the right to make one's own decisions. Fidelity is the observance of loyalty and commitment to the patient. PTS:1 DIF: Cognitive Level: Understand (Comprehension) REF: Page 99-100 | Page 103-104 TOP: Nursing Process: Planning MSC: Client Needs: Safe, Effective Care Environment

Which individual with mental illness may need emergency or involuntary admission? The individual who: a. resumes using heroin while still taking naltrexone (ReVia). b. reports hearing angels playing harps during thunderstorms. c. does not keep an outpatient appointment with the mental health nurse. d. throws a heavy plate at a waiter at the direction of command hallucinations.

ANS:D Throwing a heavy plate is likely to harm the waiter and is evidence of dangerousness to others. This behavior meets the criteria for emergency or involuntary hospitalization for mental illness. The behaviors in the other options evidence mental illness but not dangerousness. See related audience response question. PTS:1 DIF: Cognitive Level: Analyze (Analysis) REF: Page 100-101 TOP: Nursing Process: Implementation MSC: client Needs: Safe, Effective Care Environment

A client from which demographic group is most likely to have concerns about confidentiality and need reassurance from the nurse during a clinical interview? Young children Adolescents Older adults Adults who do not speak English

Adolescents All clients are concerned with confidentiality. This is especially true for adolescents, who may fear that anything they say to the nurse will be repeated to their parents. At least part of the interview should be conducted without the parent/caregiver being present. Young children are unlikely to have much awareness of confidentiality, although they may be reluctant to share information related to abuse. Older adults and clients who do not speak English may also have confidentiality concerns, but adolescents are most likely to have privacy concerns and to require reassurance. p. 78

The nurse is documenting assessment data for a newly admitted client. The nurse documents that the client demonstrates a flat affect and does not appear to be in distress despite complaints of abdominal pain. The nurse understands that this behavior is common in which culture? Asian Hispanic Native American African American

Asian In the Asian culture expression of either positive or negative emotions is a private affair; a calm facade may mask severe distress. p. 100

A military wife tearfully tells a nurse about her husband's deployment to an active war zone. This client cries daily and says to the nurse, "I am so worried that he will never come home." What response by the nurse addresses the need to identify the client's perception of the problem? "Your husband is safe. You should be proud of him rather than absorbed in worry." "Let's talk about whether crying and the feelings you describe are normal in this situation." "You will eventually get back to normal. Start doing the things that used to be fun for you." "When you find yourself starting to cry or feel sad, distract yourself by getting busy with an activity."

Asking the client to elaborate on her feelings focuses on the client's perception. This aids the client in considering other persons and events from the perspective of the client's own set of values. Telling the client her husband is safe is a false reassurance the nurse cannot know to be true. Telling the client to distract herself does not address her feelings. p. 120

Brian, a patient with schizophrenia, has been order an antipsychotic medication. The medication will likely benefit him, but there are side effects; in a small percentage of patients, it may cause a dangerous side effect. After medication teaching, Brian is unable to identify side effects and responds, "I won't have any side effects because I am iron and cannot be killed." Which response would be most appropriate under these circumstances? a. Administer the medication because Brian has made a decision to take the medication, and care should be patient centered. b. Petition the court to appoint a guardian as a substitute for Brian, as he is unable to comprehend the proposed treatment. c. Administer the medication because Brian's need for treatment is the clear priority. d. Withhold the medication until Brian is able to identify the benefits and risks of both consenting and refusing consent to the medications.

B

Sophie, aged 27 years, has a diagnosis of paranoid schizophrenia. She stopped taking her medications and believes that she is to be taken by the aliens to live with them on another planet. She was observed walking through traffic on a busy road, and then was found climbing the railing on a bridge, to "be ready for them to take me in their ship." Sophie is hospitalized. During your shift she begins running up and down the halls, banging her head on the walls, and yelling, "Get them out of my head!" On what basis can Sophie be medicated against her will? a. If Sophie has taken the medication in the past and has had no adverse effects b. If Sophie may cause imminent harm to herself or others c. If Sophie still has the capacity to make an informed decision regarding medication d. If Sophie is provided education regarding the medication before administration of the medication

B A patient may be medicated against his or her will without a court hearing in an emergency if the patient poses a danger to himself or herself or to others. The other options are not legally valid reasons to give medication against a patient's will. Cognitive Level: Apply (Application) Nursing Process: Implementation NCLEX: Safe and Effective Care Environment Text page: 103

Which ethical principle refers to the individual's right to make his or her own decisions? a. Beneficence b. Autonomy c. Veracity d. Fidelity

B Autonomy refers to self-determination, or the right to make one's own decisions. REF: 99

What ethical principle is supported when a nurse witnesses the informed consent for electroconvulsive therapy from a depressed client? a. Beneficence b. Autonomy c. Justice d. Fidelity

B Autonomy refers to self-determination. One way to exercise self-determination is to make decisions about one's care. REF: Page 99

A client who presents no danger to himself or to others is forced to take medication against his will. This situation represents a. assault. b. battery. c. defamation. d. invasion of privacy.

B Battery is the harmful, nonconsensual touching of another person. Forceful administration of medication constitutes battery. REF: 107; Table 6-3

You are working on an inpatient psychiatric unit and caring for Elizabeth, who is becoming agitated. You speak with Elizabeth one to one in a private setting, find out the reason for the agitation, and then assist Elizabeth with ways to calm down, possibly including prn medication to prevent further escalation of Elizabeth's agitation, which could lead to seclusion and/or restraints. You are making care decisions based on: a. writ of habeas corpus. b. least restrictive alternative doctrine. c. veracity. d. bioethics.

B Least restrictive alternative doctrine is described as using the least drastic means of achieving a specific goal. By doing the actions described you are possibly preventing the more restrictive setting of seclusion and/or restraints. Writ of habeas corpus is a legal term meaning a written order "to free the person." Veracity is one of the five ethical principles or guidelines. Bioethics refers to ethics in a health care setting. Cognitive Level: Apply (Application) Nursing Process: Implementation NCLEX: Safe and Effective Care Environment Text page: 100 Awarded 0.0 points out of 1.0 possible points.

The nurse is caring for a geriatric client receiving dialysis three times a week. The client frequently discusses family disputes with the nurse and often becomes verbally aggressive. Which countertransference reaction is the nurse most likely to experience toward this client? Rescue Boredom Helplessness Overinvolvement

Boredom. Due to the client's behavior, the nurse is at risk to experience a variety of countertransference reactions. If the client gives repeated uninteresting information and uses an offensive style of communication, the nurse may develop boredom and become uninterested in talking to and interacting with the client. The nurse may develop a rescue reaction (a countertransference reaction) when the client shares his or her secrets with the nurse. The nurse may develop feelings of helplessness when the treatment goals are not achieved and if the client does not participate in the treatment effectively. Overinvolvement is experienced by the nurse if the client's behavior reminds him or her of someone who is close to the nurse or of past clients. p. 112

Which rating scale is used in the evaluation and monitoring of schizophrenia? Beck Inventory Global Deterioration Scale (GDS) Brief Psychiatric Rating Scale (BPRS) Patient Health Questionnaire (PHQ-9)

Brief Psychiatric Rating Scale (BPRS) The Brief Psychiatric Rating Scale (BPRS) is used in the evaluation and monitoring of schizophrenia. The Beck Inventory and Patient Health Questionnaire (PHQ-9) are used in depression. The Global Deterioration Scale (GDS) is used to evaluate and monitor cognitive function. p. 82

A nurse makes a post on a social media page about his peer taking care of a patient with a crime-related gunshot wound during his shift in the emergency department. He does not use the name of the patient. It can be concluded that: a. the nurse has not violated confidentiality laws because he did not use the patient's name. b. the nurse cannot be held liable for violating confidentiality laws because he was not the primary nurse for the patient. c. the nurse has violated confidentiality laws and can be held liable d. the nurse cannot be held liable because posting on a social media site are excluded from confidentiality laws.

C

If a client with psychiatric illness is determined to be incompetent to make decisions affecting his care a. Staff members are required to use their best judgment when defining care. b. No treatment other than custodial care can be provided. c. The court appoints a guardian to make decisions on his behalf. d. The doctrine of least restrictive alternative is null and void.

C An incompetent client is unable to make legal decisions that would affect his care, such as consenting to surgery. A court-appointed guardian functions on behalf of the client. REF: 103-104

Which right of the client has been violated if he is medicated without being asked for his permission? a. Right to dignity and respect b. Right to treatment c. Right to informed consent d. Right to refuse treatment

C Before being given medication, the client should be fully informed about the reason for, the expected outcomes of, and any side effects of the medication. The client has the right to refuse medication. If, in a nonemergency situation, he is given medication after refusing it, his right to informed consent has been violated. REF: 100, 103-104

After the death of a client, what rule of confidentiality should be followed by nurses who provided care for the individual? a. Confidentiality is now reserved to the immediate family. b. Only HIV status continues to be protected and privileged. c. Nothing may be disclosed that would have been kept confidential before death. d. The nurse must confer with the next of kin before divulging confidential, sensitive information.

C Confidentiality extends to death and beyond. Nurses should never disclose information after the death of a client that they would have kept confidential while the client was alive.

Which of the following scenarios describe a HIPAA violation? a. Janie, the ED nurse, gives report to Amanda, a nurse on the intensive care unit, regarding Joel, who is being admitted. b. Mark, a nurse on the medical-surgical floor, calls his patient's primary care provider to obtain a list of current medications. c. Lyla, a nurse on the cardiac unit, gives report to Chloe, the nurse on the step-down unit, regarding the patient Lyla, who will be transferring, while they are walking in the hospital hallway. d. Tony, a nurse on the psychiatric unit, gives discharge information to the counseling office where his patient will be going to outpatient treatment after discharge.

C Discussing a patient's information in public places where it may be overheard is a violation of a patient's confidentiality. The other options describe appropriate interactions for patient continuity of care and support of the treatment plan by the health care team. Cognitive Level: Apply (Application) Nursing Process: Implementation NCLEX: Safe and Effective Care Environment Text page: 105

The intervention that will be most effective in preventing a nurse from making decisions that will lead to legal difficulties is a. asking a peer to review nursing intervention related decisions. b. balancing the rights of the client and the rights of society. c. maintaining currency in state laws affecting nursing practice. d. seeking value clarification about fundamental ethical principles.

C Each nurse's practice is governed by the Nurse Practice Act of the state in which the nurse practices. The nurse should always be aware of its provisions. REF: 108-109

If a client is placed in seclusion and held there for 24 hours without a written order or examination by a physician, the client has experienced a. battery. b. defamation of character. c. false imprisonment. d. assault.

C False imprisonment is the arbitrary holding of a client against his or her will. When seclusion is ordered, it is not invoked arbitrarily, but after other less restrictive measures have failed. If the client is secluded without the medical order, the measure cannot be proven as instituted for medically sound reasons. REF: Page 107-108 (Table 6-3)

If a nurse is charged with leaving a suicidal client unattended, it is being suggested that the nurse's behavior has violated the ethical principle of a. autonomy. b. veracity. c. fidelity. d. justice.

C Fidelity refers to being "true" or faithful to one's obligations to the client. Client abandonment would be a violation of fidelity. REF: 99

When the nurse reads the medical record and learns that a client has agreed to receive treatment and abide by hospital rules, the correct assumption is that the client was admitted a. per legal requirements. b. for a non-emergency. c. voluntarily. d. involuntarily.

C Voluntary admission occurs when the client is willing to be admitted and agrees to comply with hospital and unit rules. REF: Page 100-101

What are the advantages of narrative charting? Is well structured Provides consistent organization of data Can address any event or behavior Explains flow sheet findings Uses a common form of expression

Can address any event or behavior Explains flow sheet findings Uses a common form of expression Narrative charting can address any event or behavior and explain flow sheet findings. It also uses a common form of expression (narrative writing) that reduces the need to learn a system. Narrative charting is not structured and does not provide consistent organization of data. These are features of problem-oriented charting ("soapie"). Test-Taking Tip: Read the question carefully before looking at the answers: (1) Determine what the question is really asking, and look for key words; (2) read each answer thoroughly, and see if it completely covers the material the question asks; and (3) narrow the choices by immediately eliminating answers you know are incorrect. p. 88

Who determines the content and direction of the clinical interview? Nurse Client Health care provider Health care team

Client. The client should always be able to take the lead and determines the content and direction of the clinical interview, although the nurse may gently guide it by discouraging social conversation or intrusive personal questioning. The therapeutic relationship is consistently focused on the client's problem and needs. The nurse, health care provider, and health care team may take part in assisting the client's progress, but the client ultimately determines the focus of the interview. p. 110

A 75-year-old client diagnosed with dementia was found wandering a busy street and brought to the emergency department by police. What is the most effective way for the nurse to implement holistic data collection? Remain present with the client, but allow the police to conduct the primary interview. Conduct interviews with the client, the police, and appropriate family members and neighbors. Seek out an interpreter trained in conducting geriatric interviews to talk with the client and his family members. Use at least one standardized rating scale from each category of psychiatric illness to screen for all possible diagnoses.

Conduct interviews with the client, the police, and appropriate family members and neighbors. To implement holistic data collection, the nurse should conduct interviews with the client, the police, and any family members, neighbors, or other secondary sources who might have relevant information to aid in the assessment of the client's health. The nurse should prioritize the client's safety and ensure that he or she is in stable condition before the police conduct an interview, if necessary. An interpreter is unlikely to be required in this scenario. If the client has language barriers, a professional translator may be used. Only standardized rating scales relevant to this client should be used for evaluation and monitoring. p. 77

According to Rogers, which term is a synonym for genuineness? Respect Empathy Congruence Positive regard

Congruence Genuineness refers to self-awareness of one's feelings as they arise within the relationship and the ability to communicate them when appropriate. It is the ability to meet others person-to-person without hiding behind roles. Rogers uses the word "congruence" to signify genuineness. Respect, empathy, and positive regard may not work in tandem with genuineness, but they are not synonyms for genuineness. pp. 117-118

A family who is worried that an adult female might hurt herself asks for her to be admitted to the hospital. An assessment indicates moderate depression with no risk factors for suicide other than a depressed mood. The patient denies any intent or thoughts about self-harm. The family agrees that the patient has not done or said anything to suggest that she might be a danger to herself. Which of the following responses is consistent with concept of "least restrictive alternative" doctrine? a. Admit the patient as a temporary inpatient admission. b. Persuade the patient to agree to a voluntary inpatient admission. c. Admit the patient involuntarily to an inpatient mental health treatment unit. d. Arrange for an outpatient counseling appointment the next day.

D

David has an overnight pass, and he plans to spend his time with his sister and her family. As you meet with the patient and his sister just prior to the pass, the sister mentions that she has missed her brother and needs him to babysit. You notice that the patient becomes visibly agitated when she says this. How do you balance safety and the patient's right to confidentiality? a. Cancel the pass without explanation to the sister, and reschedule it for a time when babysitting would not be required of the patient. b. Suggest that the sister make other arrangements for childcare, but withhold the information the patient shared regarding his concerns about harming children. c. Speak with the patient about the safety risk involved with babysitting, seeking his permission to share this information and advising against the pass if he declines to share the information. d. Meet with the patient's sister, sharing with her the patient's previous disclosure about his anger toward children and the resultant risk that his babysitting would present.

D

What assumption can be made about the client who has been admitted on an involuntary basis? a. The client can be discharged from the unit on demand. b. For the first 48 hours, the client can be given medication over objection. c. The client has agreed to fully participate in treatment and care planning. d. The client is a danger to self or others or unable to meet basic needs.

D Involuntary admission implies that the client did not consent to the admission. The usual reasons for admitting a client over his or her objection is if the client presents a clear danger to self or others or is unable to meet even basic needs independently. REF: Page 100-101

A client reports to the nurse that once he's released he will make sure his wife will never again be able to have him committed to a psychiatric hospital. What action should the nurse take? a. None, because no explicit threat has been made. b. Ask the client if he is threathening his wife. c. Call the client's wife and report the threat. d. Report the incident to the client's therapist and document.

D The Tarasoff ruling makes it necessary for nurses to report client statements that imply the client may harm another person or persons. The nurse reports to the treatment team, and the mandated reporter (usually the professional leader of the team) is responsible for notifying the person against whom the threat was made. REF: Page 106-107

Which statement is true regarding mail sent to an involuntarily admitted client residing on a psychiatric inpatient unit? a. The client can receive mail from only family and legal sources. b. Mail must first be opened and inspected by staff. c. Receipt of mail is considered a privilege accorded the client for compliance. d. Mail is a form of social interaction and so receiving mail is a client's civil right.

D The client's civil rights are intact, despite hospitalization. The right to communicate with those outside the hospital is ensured. REF: 103-104

According to NANDA-I, which aspect of a standard nursing diagnosis is excluded in "risk for" diagnoses? Problem Etiology Supporting data Defining characteristics

Defining characteristics NANDA-I suggests that when making a "risk for" diagnosis, the diagnosis should include the risk diagnoses and risk factors (risk-related behaviors) that predispose the individual to a potential problem. Because the problem hasn't yet arisen, NANDA-I states that there can be no "related etiological factors." Test-Taking Tip: Do not worry if you select the same numbered answer repeatedly, because there usually is no pattern to the answers. p. 83

On an inpatient unit, one client assaults another client, resulting in a small laceration. Considering both clients' rights to confidentiality, how will the nurse effectively document this event? Ensure unit safety by documenting the hostile and combative characteristics of the assaulting client. Document in each client's medical record the events and actions taken, using the initials of the other client involved. Document in both clients' medical records that an occurrence (incident) report was prepared according to agency policy. Verbally report the events to other team members and minimize written documentation in order to reduce potential legal consequences.

Document in each client's medical record the events and actions taken, using the initials of the other client involved. Documenting in each record using the initials of the other client involved is the only approach that protects both clients' rights to confidentiality. Documenting the assaulting client's hostile characteristics does not address confidentiality, and the nurse should avoid defaming clients. The nurse should never document in a medical record that an occurrence report was prepared, because this is generally a privileged communication between the hospital and the hospital's attorney. Documentation is critical to protecting the nurse and other staff members, so the nurse should never minimize documentation. 89,pp. 84

A client admitted for treatment of depression after the recent death of her only child tells the nurse that she has accepted the death as God's will and is at peace with the situation. The nurse also notes during the interaction that the client is wringing her hands and not making eye contact. The nurse would interpret this as what type of message? Congruent Double-bind Nontherapeutic Culturally filtered

Double-bind A message which has incongruences between the verbal and the nonverbal aspects is considered a double-bind, or mixed, message. In this situation the message is incongruent, not congruent; thus this is incorrect. "Nontherapeutic" would not be an appropriate term to use in reference to the client's communication behaviors. Cultural filters impact the way that one interacts and listens and would not be a correct use in this situation. p. 94

As the nurse performs an assessment on a client diagnosed with breast cancer, the client says, "I will discuss my illness, but you should not share the information with anyone." Which response by the nurse is appropriate for effective nurse-client interaction? "It is a part of the assessment, and you are required to inform me of your concerns." "I assure you that I will not share the information provided by you with anyone." "I cannot maintain your secrets, because I have to follow and abide by professional ethics." "I will be sharing the information provided by you with other health care professionals but no one else."

During the assessment the nurse should inform the client that the information provided by the client will be shared with the health care professionals. This action helps safeguard the confidentiality and privacy of the client. It also helps provide for continuity of care when the client is discharged from the hospital. The statement that the client should inform the nurse of all concerns may lead to angry feelings for the client and interfere with communication; therefore, this is not an appropriate response. The nurse should not give the client false assurance by saying that the information provided by the client will not be disclosed to anyone. The nurse should not rudely deny the client's request by saying that the client's confidential medical information cannot be kept secret. This may cause the client feel rejected and is not true. pp. 111-112

A client diagnosed with prostate cancer is undergoing chemotherapy but, due to financial conditions, is unable to afford it. What is the most appropriate action by the nurse? Organize a fundraiser to pay the bills. Refer the client to another hospital or facility. Enroll the client in the financial assistance program. Educate the client about the financial assistance program.

Educate the client about the financial assistance program. It is the nurse's responsibility to help and encourage clients to use available resources. In a situation in which the client has financial issues, the nurse should inform the client about the financial assistance program. A nurse does not act for a client unless absolutely required. Paying the bill by raising funds indicates overinvolvement. Referring the client to another hospital will not solve the client's problem. By enrolling the client in the financial assistance program, the nurse would facilitate dependency. pp. 110-111

Which action by the nurse may acceptable in a social relationship but not in a therapeutic relationship? Giving advice Listening actively Clarifying feelings Giving positive regard

Giving advice. Giving and receiving advice is acceptable in a social relationship, but it is not in a therapeutic relationship. In a therapeutic relationship, it is appropriate for the nurse to assist the client in exploring alternative solutions to problems and in making his or her own decisions. Actively listening is imperative for a therapeutic relationship so this is incorrect. Clarifying feelings is appropriate, as is giving positive regard. p. 110

An older adult client's spouse died 2 months ago. Since then, he has stopped bathing and changing his clothes regularly. He has expressed to the nurse that he is lonely and doesn't wish to live without his spouse. What nursing intervention is most appropriate for this client? Use physical restraints to help the client refrain from self-harm. Search the client and personal belongings for weapons daily. Place the client in seclusion to be sure his environment contains nothing he can use to self-harm. Help the client identify a support network of friends, family, and care providers.

Help the client identify a support network of friends, family, and care providers. The most appropriate intervention is to help the client identify a support network of friends, family, and care providers. Using physical restraints is only appropriate when the client is in immediate danger and all other less restrictive interventions have been tried first. Searching the client for weapons daily is unlikely to be necessary for the client at present. Placing the client in seclusion may make him feel more isolated, and the client should always be maintained in the least restrictive environment. Test-Taking Tip: Multiple choice questions can be challenging, because students think they will recognize the right answer when they see it or that the right answer will somehow stand out from the other choices. This is a dangerous misconception. The more carefully the question is constructed, the more each of the choices will seem like the correct response. p. 85

Patients from which culture most likely use dramatic body language when discussing emotional problems? Asian Americans African Americans Hispanic Americans British Americans

Hispanic Americans Hispanic Americans may use dramatic body language when describing emotional problems. Asian, African, and British Americans are less likely to use dramatic body language when describing emotional problems. p. 100

The nurse is interviewing a client who immigrated from India. The nurse asks questions about the client's home culture and religious practices in addition to questions about medical history. What does this combination of questions most represent? Holistic approach to care Performance-based learning Milieu management Evidence-based practice

Holistic approach to care The holistic approach to care views the client as a complex blend of many parts. It involves awareness of psychological, social, cultural, environmental, functional, and spiritual issues as well as ethnicity, sexual orientation, and age. The use of clinical simulations is an example of performance-based learning. Milieu management includes orienting clients to their rights and responsibilities and informing them in a culturally competent manner about the need for structure, maintenance of a safe environment, and limits set on the unit. Evidence-based practice integrates the best current evidence with clinical practice to deliver optimal health care. p. 77

While assessing a client diagnosed with sickle cell anemia, the nurse tries to develop trust and establish rapport with the client. Which nursing intervention is appropriate for this phase of nurse-client interaction? Promote problem-solving skills. Teach alternative ways of expressing feelings. Inform the client about the scheduled meeting. Share the feelings of helplessness with colleagues and health care provider.

Inform the client about the scheduled meeting. During the orientation phase, the nurse tries to develop trust and establish rapport with the client. The nurse schedules an interview and informs the client of the time, date, and duration of the meeting. During the working phase, the nurse promotes problem-solving skills and teaches alternative ways to express feelings. Working with the client to accommodate his or her schedule indicates the concern of the nurse toward the client and encourages the client to express his or her thoughts and feelings to the nurse. In the preorientation phase, the nurse takes the advice of colleagues and the health care provider before interacting with the client. pp. 115-116

A Native American adult is hospitalized. The emergency department assessment indicates auditory and visual hallucinations. The client states, "My dead father told me to kill myself to save me from the bad spirits." What would be an appropriate nursing intervention for the nursing care plan? -Initiate a consultation between the hospital chaplain and the client. -Provide the client with frequent periods alone for meditation and prayer. -Assign only Native American staff members to provide this client's care. -Consult the family, with the client's consent, for a spiritual healer from the client's tribe.

Initiate a consultation between the hospital chaplain and the client. Culturally competent care is a holistic approach that addresses the mind-body-spiritual aspect of individuals. Consulting with a spiritual leader may help address the client's spiritual concerns. It is unsafe to leave a potentially suicidal client alone. It may not be possible to only assign Native American staff members to care for this client, and all staff members should be culturally respectful and knowledgeable. The client is not mentally well enough to provide consent to allow the nurse to speak to the client's family. p. 100

The nurse is assessing a 9-year-old child. The child's parent reports that the child wakes up several times throughout the night crying. The nurse finds that the child has had multiple fractures over the past year. What is the nurse's most appropriate action? Stop the interview, and use observation of the child at play as the main source of information. Avoid using any secondary sources for data collection, because doing so would violate the child's confidentiality. Interview the child separately from the parents, so the child is less reluctant to give details about possible physical abuse. Ask the child to leave the room for the remainder of the interview, because the parents are the best source for determining the child's inner feelings.

Interview the child separately from the parents, so the child is less reluctant to give details about possible physical abuse. Multiple fractures over the past year, along with chronic nightmares, may be suggestive of child abuse. The nurse should interview the child separately from the parents, so the child is less reluctant to give details about possible physical abuse. Assessment of children should be accomplished by a combination of interview and observation. Secondary sources include family, friends, neighbors, and others with knowledge about the child. They can be invaluable sources of data about the child, and such data can be gathered without violating the child's confidentiality. Although the parents are an important source of information, the child is the best source for determining his or her own inner feelings and emotions. pp. 77-78

When completing an assessment on a client who presented to the emergency room with extreme anxiety, which assessment finding would the nurse characterize as a nonverbal autonomic physiological response? Is very pale Appears very thin Rarely smiles; limited facial expressions Tapping fingers restlessly on the bedside table

Is very pale Paleness is an observable autonomic physiologic nonverbal behavior. Thin appearance is a physical characteristic. Smiling is considered a nonverbal facial expression behavior. Tapping the fingers is a body movement and thus an aspect of nonverbal body behaviors. Test-Taking Tip: Identifying content and what is being asked about that content is critical to your choosing the correct response. Be alert for words in the stem of the item that are the same or similar in nature to those in one or two of the options . p. 94

When conducting a clinical interview of a client, which techniques should the nurse implement to facilitate effective communication? Select all that apply. Maintaining silence to encourage the client to talk Making nonthreatening observations to initiate conversation Making value judgments about the client's behaviors Reassuring the client that everything will be all right Probing the client for relevant information Asking open-ended questions

Maintaining silence to encourage the client to talk Asking open-ended questions Making nonthreatening observations to initiate conversation Maintaining silence, making observations, and asking open-ended questions are effective therapeutic techniques. Maintaining silence when necessary gives the client time to recollect and think. Making observations and calling attention to the client's behavior helps start a conversation with a withdrawing client. Asking open-ended questions promotes long conversations and encourages the client to respond fully. Making value judgments prevents problem solving, and the client may get angry or dissatisfied. False reassurances will make the client feel unimportant. Probing the client may make him or her withdrawal if the nurse pushes for conversations the client is not ready to have. p. 120

Informing clients about their rights and responsibilities is an aspect of which nursing intervention? Milieu therapy Integrative therapy Coordination of care Health teaching and promotion

Milieu therapy Milieu therapy includes informing clients about their rights and responsibilities, as well as the need for structure, maintenance of a safe environment, and limits set on the unit. Integrative therapy includes interventions that complement pharmacological and biological therapies. Health teaching and promotion includes teaching coping skills, self-care activities, stress management, problem-solving skills, relapse prevention, and conflict management. p. 86

While interacting with a client involved in an automobile accident involving injuries, the nurse states: "I know you are upset but it could have been far worse, so let's focus on taking care of you for now." Which nontherapeutic communication did the nurse employ? False reassurance Minimizing feelings Making a value judgment Giving premature advice

Minimizing feelings Stating that the accident could have been far worse minimizes the client's feelings and indicates the nurse does not empathize with the client. False reassurance underrates the client's feelings. Value judgments may cause the client to feel misunderstood. Giving premature advice assumes the nurse knows best. p. 99

Which technology holds strong promise for treatment of mental health for psychiatric clients? Big data Facebook Telehealth Mobile apps

Mobile apps Thousands of mobile apps exist which address psychiatric conditions and according to surveys, psychiatric clients are willing to use apps to monitor their mental health. Big data, Facebook, and teleheath are less helpful in the treatment of mental health for psychiatric clients. Test-Taking Tip: Once you have decided on an answer, look at the stem again. Does your choice answer the question that was asked? If the question stem asks "why," be sure the response you have chosen is a reason. If the question stem is singular, then be sure the option is singular, and the same for plural stems and plural responses. Many times, checking to make sure that the choice makes sense in relation to the stem will reveal the correct answer. pp. 91, 101-102

Which nursing action demonstrates the ability to engage in active listening during a nurse-client conversation? Noting that the client is wringing his or her hands nervously Restating in different words the content of the client's message Introducing new topics when the conversation reverts to silence Sharing with the client that they seem as if he or she is disappointed

Noting that the client is wringing his or her hands nervously Noting that the client is wringing the hands nervously is correct because the nurse is actively listening by observing nonverbal behavior. Restating in different words the content of the client's message is incorrect because it is paraphrasing, which is a clarifying technique in communication. Introducing new topics during silence is incorrect, because it does not demonstrate active listening. Sharing with the client that he or she seems disappointed is incorrect because it is demonstrating reflecting, which is a clarifying technique. p. 95

What are the disadvantages of problem-oriented charting? Limits entries to problems Commonly results in inclusion of unnecessary information Requires time and effort to structure the information Frequently leads to omission of elements of the nursing process May result in loss of data about progress

Problem-oriented charting limits entries to problems, requires time and effort to structure the information, and may result in loss of data about progress. Disadvantages of narrative charting include that it commonly results in inclusion of unnecessary information and frequently leads to omission of elements of the nursing process. p. 88

Which would be considered a cognitive factor that could impact accurate transmission or interpretation of messages? Response to stress Problem-solving ability Cultural differences Relationship misunderstandings

Problem-solving ability Problem-solving abilities are considered cognitive factors which can impede accurate transmission or interpretation of messages. Response to stress and relationship misunderstandings are considered emotional factors. Cultural differences are considered social factors. p. 92

As of the late 1990s, the Institute of Medicine found the United States lacking in which areas of health care? Select all that apply. Access to a broad range of specialists A sufficient number of licensed nurses Quality care across geographic locations Services rooted in evidence-based practices Respectful, responsible, and patient-centered care

Quality care across geographic locations Services rooted in evidence-based practices Respectful, responsible, and patient-centered care Reports from the Institute of Medicine (IOM) found the American health care system lacking in a number of areas, including quality care across geographic locations, services based on evidence-based practice, and respectful and responsible care that is patient centered. These reports did not find access to specialists or number of licensed nurses to be lacking. p. 76

A 60-year-old client reports, "I stopped taking my medication shortly after it became unsafe for me to drive due to my health problems." What is the most appropriate action by the nurse? Have the client sign a release form to protect against malpractice lawsuits. Question the client about difficulty obtaining the medication. Contact the appropriate license-issuing department to request the client's license be reissued. Make a note in the client's chart that the client was "nonadherent" with medication instructions.

Question the client about difficulty obtaining the medication. When a client does not follow medication or treatment plans, the nurse should find out what is going on in the client's life to challenge his or her adherence. The nurse should help the client brainstorm solutions to the challenges that come up. The best way to protect against malpractice is to meticulously document rationale for treatment, offer clear explanations what the client should do, and note whether the client actually complied with that advice. The nurse should not request that the client's license be reissued if the client presents a danger behind the wheel. Although using the term "nonadherent" is preferred over "noncompliant," simply making a note of this in the chart is less appropriate that having an extended conversation about the issue with the client. The nurse should not have a client sign a release form in this instance. p. 87

What phrase should the nurse include in all nursing diagnoses related to health promotion? Risk for Willingness to Coping skills Readiness for enhanced

Readiness for enhanced Health promotion diagnoses are always stated in the form of "readiness for enhanced" and supported by the data/defining characteristics. "Risk for" is not relevant to a health promotion diagnosis. "Willingness to" and "coping skills" may be included in some, but not all, health promotion diagnoses when relevant. p. 83

Which intervention will assist the nurse in improving his or her communication and interviewing skills? Observing other clinicians during interviews. Reviewing written records of the nurse-client session. Interacting with multiple clients. Asking each client more questions.

Reviewing written records of the nurse-client session. The best way to improve skills is to review clinical interactions exactly as they occurred. Written records of a nurse-client session are called process recordings. These records help in reviewing clinical interactions with the client and help the student nurse identify patterns in communication. Process recordings consist of both verbal and nonverbal communication between the client and the nurse. Having an observing clinician can distract the concentration of both the client and the nurse. Interacting with multiple clients can help to some extent, but without proper documentation and analysis of patterns, the student nurse will not learn as much as he or she would from the process recordings. Asking too many questions of the client will make the client annoyed and is not good practice for effective communication and interviewing. p. 121

A 55-year-old lives 100 miles from her parents and mother-in-law. In the past year, her father had back surgery, her mother broke her hip, and her mother-in-law had a cardiac event. Which nursing diagnosis is most applicable to the 55-year-old? Risk for complicated grieving related to impending deaths of parents Risk for injury related to frequent long drives to care for aging parents Risk for chronic low self-esteem related to overwhelming responsibilities Risk for caregiver role strain related to responsibilities for care of aging parents

Risk for caregiver role strain related to responsibilities for care of aging parents Risk diagnoses are based on the high probability that an event will occur in a vulnerable person. The diagnosis must be supported by the risk factors that predispose the individual to the event. In this case, the distance and the recent health problems the individual's family members have experienced predispose her to caregiver role strain as her responsibilities increase. None of the family members have a terminal illness, so it is not known if their deaths are impending. Injury related to long drives is unlikely and therefore not a risk factor. There is no evidence that the woman's self-esteem is being affected by her responsibilities, so this is not a risk factor.

A client telephones the nurse at the mental health center daily, giving lengthy details about multiple somatic complaints and relationship problems. Which limit-setting strategy should the nurse employ? Suggest the client call other people in the community. Say to the client, "I can talk to you for 15 minutes twice a week." Use the telephone's caller identification to screen calls from the client. Tell the client, "You should discuss these concerns with your personal physician rather than me."

Say to the client, "I can talk to you for 15 minutes twice a week." Telling the client that the nurse can be available for 15 minutes twice a week sets a clear, absolute limit that cannot be misinterpreted. Suggesting the client call other people does not address the nurse's boundaries in the relationship. Using caller identification to avoid the client does not communicate a limit. Unless the client's concerns are out of the nurse's scope, it is unnecessary for the nurse to defer to the physician. p. 111

Which rating scale is used in the evaluation and monitoring of schizophrenia? Beck Inventory Zung Self-Report Inventory Global Deterioration Scale (GDS) Scale for Assessment of Negative Symptoms (SANS)

Scale for Assessment of Negative Symptoms (SANS) The Scale for Assessment of Negative Symptoms (SANS) is used in the evaluation and monitoring of schizophrenia. The Beck Inventory and Zung Self-Report Inventory are used in depression. The Global Deterioration Scale (GDS) is used to evaluate and monitor cognitive function. Test-Taking Tip: Sometimes the reading of a question in the middle or toward the end of an exam may trigger your mind with the answer or provide an important clue to an earlier question. p. 82

The nurse in a reproductive clinic is completing an assessment on a teen who has presented with possible signs of a sexually transmitted disease (STD). The teen is blushing, not making eye contact, and seems embarrassed by the current situation. Which mode of communication may be most effective initially with this client? Silence Focusing Giving information Verbalizing the implied

Silence In this case, the client may feel embarrassed about having an STD. Silence can be an effective form of communication and may be initially useful to allow the client time to share thoughts and feelings that otherwise may be withheld. Focusing can be useful when a client jumps from topic to topic, which is not occurring in this situation. Giving information is used when a client needs facts or knowledge about care or the role of the nurse, and would not be most effective at the current point in this interaction. Verbalizing what is being implied helps put into concrete terms what the client has implied; in this case the client has not yet spoken so this technique would not be appropriate. p. 95

The family member of a client being discharged needs information concerning a newly prescribed medication for the client and wants to ask the nurse about the medication. This would be considered which aspect of Berlo's communication model? -Media -Message -Stimulus -Feedback

Stimulus A stimulus, such as a need for information, comfort, or advice, stimulates communication. Media refers to the variety of ways a message can be sent. Message is the information sent or expressed. Feedback is the response of the receiver of the message to the sender. p. 92

The family member of a client being discharged needs information concerning a newly prescribed medication for the client and wants to ask the nurse about the medication. This would be considered which aspect of Berlo's communication model? Media Message Stimulus Feedback

Stimulus A stimulus, such as a need for information, comfort, or advice, stimulates communication. Media refers to the variety of ways a message can be sent. Message is the information sent or expressed. Feedback is the response of the receiver of the message to the sender. p. 92

Which factors should be considered to ensure a plan of care is patient-centered? Select all that apply. The most recent and relevant scientific research The available community resources and technology The client's capabilities given age, physical strength, and condition The client's willingness to change The client's preferences, health practices, and goals

The available community resources and technology The client's capabilities given age, physical strength, and condition The client's willingness to change The client's preferences, health practices, and goals When planning care, the nurse should consider the client's capabilities given his or her age, physical strength, condition, and willingness to change; the client's preferences, health practices, coping styles, developmental level, and recovery goals (to name but a few); and the actual available community resources and technology. The most recent and relevant scientific research should be considered to ensure that care is evidence based but is not specific to the patient-centered approach. p. 85

Of what must the nurse first be aware in order to help a client develop his or her resources? Resistance Countertransference The client's strengths Negative transference

The client's strengths. Nurses work to bolster a client's strengths, to identify areas of dysfunction, and to assist in the development of new coping strategies. While being aware of a client's resistance, countertransference, and negative transference is important, the first step is identifying the client's strengths. Test-Taking Tip: You have at least a 25% chance of selecting the correct response in multiple choice items. If you are uncertain about a question, eliminate the choices that you believe are wrong and then call on your knowledge, skills, and abilities to choose from the remaining responses. p. 119

Which seating arrangement appropriately supports communication during a nurse-client interview? The nurse behind a desk and the client in a chair in front of the desk The nurse and client sitting at a 90-degree angle to each other The client sitting in a chair and the nurse standing a few feet away The nurse and client sitting in chairs while facing each other

The nurse and client sitting at a 90-degree angle to each other Sitting at a 90-degree angle to each other is less intense and allows the client and nurse to look away from each other without discomfort. In these positions, the two are at the same height, so neither is in an inferior position. The nurse should avoid placing a barrier like a desk between him- or herself and the client. If the nurse stands while the client sits, this may be threatening to the client. Face-to-face seating is a more confrontational arrangement and therefore can produce anxiety. p. 119

Which behavior by the nurse most clearly demonstrates promoting a social relationship instead of a therapeutic nurse-client relationship? The nurse shares stories about his or her children. The nurse talks about favorite television shows with the client. The nurse confides in the client about a frustrating day at work. The nurses talks about a client with another nurse outside the workplace.

The nurse confides in the client about a frustrating day at work. In order to maintain a therapeutic nurse-client relationship, the nurse must remember to focus all interactions on the client and the client's needs. A nurse who talks about a frustrating day at work is focusing on his or her own needs, which imparts a social relationship rather than a therapeutic relationship. A nurse can share a brief story about his or her own personal life, as long as the conversation and interaction remains focused on the client. The nurse can be friendly with the client and talk about favorite television shows while still maintaining a therapeutic relationship. The nurse should not talk about clients outside of the workplace due to privacy issues, but this inappropriate nurse-to-nurse interaction does not necessarily affect the nurse's therapeutic relationship with the client. Test-Taking Tip: Sometimes the reading of a question in the middle or toward the end of an exam may trigger your mind with the answer or provide an important clue to an earlier question. p. 110

A nurse is assessing a Japanese client. While talking to the client, the nurse maintains direct eye contact. What might the client interpret from the nurse's behavior? The nurse is showing disrespect to the client. The nurse is being attentive to the client. The nurse is showing respect to the client. The nurse does not understand what the client says.

The nurse is showing disrespect to the client. Maintaining direct eye contact is considered to be a sign of rudeness and arrogance in some Asian cultures. On the other hand, in the United States, direct eye contact indicates attentiveness and respect. Although the nurse may intend to communicate attentiveness and respect, the client is unlikely to interpret the eye contact this way. Eye contact is unlikely to communicate a lack of understanding. Test-Taking Tip: Read the question carefully before looking at the answers: (1) Determine what the question is really asking, and look for key words; (2) read each answer thoroughly, and see if it completely covers the material the question asks; and (3) narrow the choices by immediately eliminating answers you know are incorrect. p. 100

A nurse plans to interview a hospitalized client who is lying supine with the head elevated at 45 degrees. Which initial action by the nurse will most enhance the probability of achieving a therapeutic interaction? The nurse should transfer the client to a chair near the door if the client is able to move. The nurse should stand during the interview to establish professional credibility. The nurse should select a chair or stool that positions the nurse at about the same level as the client. The nurse should maintain the room arrangement without alteration to enhance the client's comfort.

The nurse should select a chair or stool that positions the nurse at about the same level as the client. The nurse should arrange the setting to maximize communication. In all settings, chairs should be arranged so that conversation can take place in normal tones of voice and so that eye contact can be comfortably maintained or avoided. Seating should use the same height whenever possible to support the therapeutic relationship. Placing the client near the door may jeopardize the nurse's safe exit if it should become necessary. The nurse should not stand or otherwise be positioned above the client, because this may make the client feel uncomfortable or inferior. The nurse should change the arrangement of the furniture if necessary to maximize the therapeutic value of the visit. p. 119

A client diagnosed with major depressive disorder has been socially isolated. The nurse invites the client to a staff luncheon to honor the supervisor. Which analysis best applies to this scenario? The invitation supports development of the client's self-esteem. The nurse's action blurs the boundaries of a therapeutic relationship. The nurse's invitation exposes the client to a therapeutic social activity. The invitation provides an opportunity for the client to practice interactions with others.

The nurse's action blurs the boundaries of a therapeutic relationship. The nurse-client relationship should be conducted within appropriate and clear boundaries. In this scenario, the nurse's invitation blurs those boundaries by adding a social dimension. Supporting the client's self-esteem, exposing the client to a therapeutic social activity, and providing the client an opportunity to interact with others move the relationship toward a social, rather than therapeutic, one. Test-Taking Tip: Do not worry if you select the same numbered answer repeatedly, because there usually is no pattern to the answers. p. 110

When a nurse and client meet and have a limited but helpful relationship, what can the relationship be called? Crisis intervention Therapeutic encounter Autonomous interaction Preorientation phenomenon

Therapeutic encounter A therapeutic encounter is a short but helpful interaction between the nurse and client. Crisis intervention, autonomous interaction, and preorientation phenomenon are not instances of a nurse and client meeting informally or having an otherwise limited but helpful relationship. Test-Taking Tip: Avoid looking for an answer pattern or code. There may be times when four or five consecutive questions have the same letter or number for the correct answer. p. 112

What is one purpose of the psychosocial assessment? To identify stressors and coping mechanisms To evaluate current cognitive processes To differentiate physical- and somatic-based symptoms To monitor severity of depression

To identify stressors and coping mechanisms Identification of stressors and coping mechanisms occurs during the psychosocial assessment. Evaluating current cognitive processes is the main purpose of the mental status evaluation (MSE). Thorough physical and psychological health evaluations will differentiate physical- and somatic-based symptoms. A number of standardized rating scales are used to evaluate and monitor the severity of depression. p. 80

What is the main purpose of the mental status evaluation (MSE)? To monitor severity of depression To assess current cognitive processes To identify stressors and coping mechanisms To differentiate physical- and somatic-based symptoms

To assess current cognitive processes The main purpose of the MSE is to assess current cognitive processes. A number of standardized rating scales are used to evaluate and monitor the severity of depression. Identification of stressors and coping mechanisms occurs during the psychosocial assessment. Thorough physical and psychological health evaluations will differentiate physical- and somatic-based symptoms. Test-Taking Tip: Avoid looking for an answer pattern or code. There may be times when four or five consecutive questions have the same letter or number for the correct answer. p. 80

What is the primary goal of the Health Insurance Portability and Accountability Act (HIPAA)? To prepare nurses with the knowledge, skills, and attitudes required to enhance quality, care, and safety in the health care setting To ensure that an individual's health information is properly protected while receiving high-quality health care To assess all clients for suicide risk and prioritize the safety of clients at risk for suicide To inform clients in a culturally competent manner about the need for structure, safety, and quality of care

To ensure that an individual's health information is properly protected while receiving high-quality health care The primary goal of HIPAA is to ensure that an individual's health information is properly protected while receiving high-quality health care. The primary goal of Quality and Safety Education for Nurses (QSEN) is to prepare nurses with the knowledge, skills, and attitudes required to enhance quality, care, and safety in the health care setting. It is important for nurses to assess all clients for suicide risk and prioritize the safety of clients at risk for suicide. Milieu therapy involves informing clients in a culturally competent manner about the need for structure, safety, and quality of care. p. 77

What is the primary goal of Quality and Safety Education for Nurses (QSEN)? To assess all clients for suicide risk and prioritize the safety of clients at risk for suicide To inform clients in a culturally competent manner about the need for structure, safety, and quality of care To ensure that an individual's health information is properly protected while receiving high-quality health care To prepare nurses with the knowledge, skills, and attitudes required to enhance quality, care, and safety in the health care setting

To prepare nurses with the knowledge, skills, and attitudes required to enhance quality, care, and safety in the health care setting The primary goal of QSEN is to prepare nurses with the knowledge, skills, and attitudes required to enhance quality, care, and safety in the health care setting. It is important for nurses to assess all clients for suicide risk and prioritize the safety of clients at risk for suicide. Milieu therapy involves informing clients in a culturally competent manner about the need for structure, safety, and quality of care. The primary goal of the Health Insurance Portability and Accountability Act (HIPAA) is to ensure that an individual's health information is properly protected while receiving high-quality health care. p. 76

What is the goal of a nurse's self-awareness assessment? To understand how bias can distort the nurse's understanding of the client To eliminate personal biases and avoid bringing emotion to the clinical setting To become familiar with the customs and practices of other cultures and religions To identify personal "off days" and ask a colleague to cover the nurse's shifts when they occur

To understand how bias can distort the nurse's understanding of the client We all have personal biases, and the nurse should understand how his or her own biases can distort his or her understanding of clients. It is impossible to eliminate personal biases, and some degree of emotion is appropriate in the clinical setting to provide compassionate care. Becoming familiar with different cultures and religions is welcome, but the nurse's personal biases will persist in the presence of this knowledge. Nurses should learn how to work through their "off days" without interfering with providing quality care. p. 82

What is the goal of a nurse's self-awareness assessment? To understand how bias can distort the nurse's understanding of the client To eliminate personal biases and avoid bringing emotion to the clinical setting To become familiar with the customs and practices of other cultures and religions To identify personal "off days" and ask a colleague to cover the nurse's shifts when they occur

To understand how bias can distort the nurse's understanding of the client We all have personal biases, and the nurse should understand how his or her own biases can distort his or her understanding of clients. It is impossible to eliminate personal biases, and some degree of emotion is appropriate in the clinical setting to provide compassionate care. Becoming familiar with different cultures and religions is welcome, but the nurse's personal biases will persist in the presence of this knowledge. Nurses should learn how to work through their "off days" without interfering with providing quality care. p. 82

What characteristics define client outcomes? Select all that apply. Variable Identified prior to the nursing diagnosis Measurable Reflective of client's actual state Selected from a NANDA-I list

Variable Measurable Reflective of client's actual state Outcomes are variable, measurable, and stated in terms that reflect a client's actual state. They are informed by the nursing diagnosis, which must be determined first. NANDA-I lists nursing diagnoses, not client outcomes. p. 88

What characteristics define client outcomes? Variable Identified prior to the nursing diagnosis Measurable Reflective of client's actual state Selected from a NANDA-I list

Variable Measurable Reflective of client's actual state Outcomes are variable, measurable, and stated in terms that reflect a client's actual state. They are informed by the nursing diagnosis, which must be determined first. NANDA-I lists nursing diagnoses, not client outcomes. p. 88

In which nurse-client interaction would it be most appropriate for the nurse to consider using touch? -Comforting a tearful client of Japanese heritage -Counseling a child who was physically abused by a parent -Welcoming a person of Hispanic heritage to a new group session -Interacting with a Native American who has a hearing impairment

Welcoming a person of Hispanic heritage to a new group session Therapeutic use of touch is a basic aspect of the nurse-client relationship and often perceived as a gesture of warmth and friendship, but the response to touch is culturally defined. Many Hispanic Americans are accustomed to frequent physical contact and perceive it in a positive way. In Japanese culture, touch is generally limited. A child who has been physically abused may not know how to interpret the nurse's touch, and the facility may even have a "no-touch" policy in such cases. As in Japanese culture, touch in Native American culture is generally limited. p. 101


Set pelajaran terkait

IB Computer Science Case Study 2018

View Set

Chapter 1 - Introduction to Criminal Behavior

View Set

Distribution System Passive Devices

View Set